Формула скорости с постоянным ускорением: Движение тела с постоянным свободного падения – уравнение, формула

Содержание

1. График зависимости скорости от времени при прямолинейном движении с постоянным ускорением

Самое простое из всех неравномерных движений — это прямолинейное движение с постоянным ускорением.

 

При движении с постоянным ускорением (a→=const→) скорость тела линейно зависит от времени:

 

v→=v→o+a→t.

 

В проекциях на ось \(Ox\) данные равенства имеют вид:

 

ax=const;

 

vx=vox+axt.

 

Построим  и ax<0.

Примем vox>0.

 

Поскольку в обоих случаях ax=const, то графиком зависимости axt ускорения от времени в обоих случаях будет прямая, параллельная оси времени.

Только при ax>0 данная прямая будет лежать в верхней полуплоскости (рис. \(1\)), а при ax<0 — в нижней (рис. \(2\)).

 

  

Рис. \(1\). График зависимостей axt и vxt, для случая ax>0

 

 

Рис. \(2\). График зависимостей axt и vxt, для случая ax<0

 

Графиком зависимости скорости движения тела от времени vxt является прямая, пересекающая ось скорости в точке v0 и образующая с положительным направлением оси времени острый угол при ax>0 (рис. \(3\)) и тупой угол при ax<0 (рис. \(4\)).

 

  

Рис. \(3\). График зависимости скорости движения тела от времени vxt  

 

  

Рис. \(4\). График зависимости скорости движения тела от времени vxt, проекция vx скорости тела вначале положительна

 

График на рисунке \(3\) описывает возрастание проекции скорости vx. При этом модуль скорости тела также растёт. Данный график соответствует равноускоренному движению тела.

 

График на рисунке \(4\) показывает, что проекция vx скорости тела вначале положительна.

Она уменьшается и в момент времени t=tп становится равной нулю.

В этот момент тело достигает точки поворота, в которой направление скорости тела меняется на противоположное, и при t>tп проекция скорости становится отрицательной.

 

Из последнего графика также видно, что до момента поворота модуль скорости уменьшался — тело двигалось равнозамедленно.

При t>tп модуль скорости растёт — тело движется равноускоренно.

Для любого равнопеременного прямолинейного движения площадь фигуры между графиком vx и осью времени \(t\) численно равна проекции перемещения Δrx.

 

Рис. \(5\). Трапеция, образовываемая осями координат и графиком 

 

Согласно данному правилу, проекция перемещения Δrx при равнопеременном движении определяется площадью трапеции \(ABCD\) (рис. \(5\)). Эта площадь равна полусумме оснований трапеции, умноженной на её высоту:

  

S=AB+DC2⋅AD.

  

В результате:

  

Δrx=vox&plus;vx2⋅Δt.

  

Из данной формулы получим формулу для среднего значения проекции скорости:

  

vxср=ΔrxΔt=vox&plus;vx2.

  

При движении с постоянным ускорением данное отношение выполняется не только для проекций, но и для векторов скорости:

  

vcp→=vo→&plus;v→2.

Средняя скорость движения с постоянным ускорением равна полусумме начальной и конечной скоростей.

Источники:

Рис. 1. График зависимостей axt и vxt, для случая ax>0. © ЯКласс.

Рис. 2. График зависимостей axt и vxt, для случая ax<0. © ЯКласс.

Рис. 3. График зависимости скорости движения тела от времени vxt. © ЯКласс.

Рис. 4. График зависимости скорости движения тела от времени vxt, проекция vx скорости тела вначале положительна. © ЯКласс.

Рис. 5. Трапеция, образовываемая осями координат и графиком. © ЯКласс. 

«Ускорение. Прямолинейное движение с постоянным ускорением»

На данном уроке, тема которого: «Уравнение движения с постоянным ускорением. Поступательное движение», мы вспомним, что такое движение, каким оно бывает. Также вспомним, что такое ускорение, рассмотрим уравнение движения с постоянным ускорением и как им пользоваться для определения координаты движущегося тела. Рассмотрим пример задачи для закрепления материала.

Главная задача кинематики – определить положение тела в любой момент времени. Тело может покоиться, тогда его положение меняться не будет (см. рис. 1).

Рис. 1. Покоящееся тело

Тело может двигаться прямолинейно с постоянной скоростью. Тогда его перемещение будет изменяться равномерно, то есть одинаково за равные промежутки времени (см. рис. 2).

Рис. 2. Перемещение тела при движении с постоянной скоростью

Перемещение , скорость, умноженная на время, это мы давно умеем делать. Тело может двигаться с постоянным ускорением, рассмотрим такой случай (см. рис. 3).

Рис. 3. Движение тела с постоянным ускорением

Ускорение

Ускорение – это изменение скорости за единицу времени (см. рис. 4):

Рис. 4. Ускорение

Скорость – векторная величина, поэтому и изменение скорости, т. е. разность векторов конечной и начальной скорости, является вектором. Ускорение – тоже вектор, направленный туда же, куда и вектор разности скоростей (см. рис. 5).

Мы рассматриваем прямолинейное движение, поэтому можно выбрать координатную ось вдоль прямой, вдоль которой происходит движение, и рассматривать проекции векторов скорости и ускорения на эту ось:

Тогда равномерно изменяется его скорость: (если его начальная скорость была равна нулю). Как теперь найти перемещение? Скорость умножить на время – нельзя : скорость постоянно менялась; какую брать? Как определить, где при таком движении будет находиться тело в любой момент времени – сегодня мы эту проблему решим.

Сразу определимся с моделью: мы рассматриваем прямолинейное поступательное движение тела. В таком случае можем применять модель материальной точки. Ускорение направлено вдоль той же прямой, вдоль которой материальная точка движется (см. рис. 6).

Поступательное движение

Поступательное движение – это такое движение, при котором все точки тела движутся одинаково: с одинаковой скоростью, совершая одинаковое перемещение (см. рис. 7).

Рис. 7. Поступательное движение

А как еще может быть? Взмахните рукой и проследите: понятно, что ладонь и плечо двигались по-разному. Посмотрите на колесо обозрения: точки вблизи оси почти не движутся, а кабинки движутся с другой скоростью и по другим траекториям (см. рис. 8).

Рис. 8. Движение выбранных точек на колесе обозрения

Посмотрите на движущийся автомобиль: если не учитывать вращение колес и движение частей мотора, все точки автомобиля движутся одинаково, движение автомобиля считаем поступательным (см. рис. 9).

Рис. 9. Движение автомобиля

Тогда нет смысла описывать движение каждой точки, можно описать движение одной. Автомобиль считаем материальной точкой. Обратите внимание, что при поступательном движении линия, соединяющая любые две точки тела при движении, остается параллельной сама себе (см. рис. 10).

Рис. 10. Положение линии, соединяющей две точки

Автомобиль ехал прямолинейно в течение часа. В начале часа его скорость была 10 км/ч, а в конце – 100 км/ч (см. рис. 11).

Рис. 11. Рисунок к задаче

Скорость изменялась равномерно. Сколько километров проехал автомобиль?

Проанализируем условие задачи.

Скорость автомобиля изменялась равномерно, то есть всё время пути его ускорение было постоянным. Ускорение по определению равно:

Автомобиль ехал прямолинейно, поэтому мы можем рассматривать его движение в проекции на одну ось координат:

Найдем перемещение.

Пример возрастающей скорости

На стол кладут орехи, по одному ореху в минуту. Понятно: сколько минут пройдет, столько орехов на столе окажется. А теперь представим, что скорость накладывания орехов равномерно возрастает с нуля: первую минуту орехов не кладут, во вторую кладут один орех, потом два, три и так далее. Сколько орехов окажется на столе через какое-то время? Понятно, что меньше, чем если бы максимальная скорость поддерживалась всегда. Причем хорошо видно, что меньше в 2 раза (см. рис. 12).

Рис. 12. Количество орехов при разной их скорости выкладывании

Так же и с равноускоренным движением: допустим, сначала скорость была равна нулю, в конце стала равна (см. рис. 13).

Рис. 13. Изменение скорости

Если бы тело постоянно двигалось с такой скоростью, его перемещение было бы равно , но поскольку скорость равномерно возрастала – то в 2 раза меньше.

Мы умеем находить перемещение при РАВНОМЕРНОМ движении: . Как обойти эту проблему? Если скорость изменяется не на много, то движение можно приближенно считать равномерным. Изменение скорости будет небольшим за небольшой интервал времени (см. рис. 14).

Рис. 14. Изменение скорости

Поэтому разобьем время в пути T на N небольших отрезков длительностью (см. рис. 15).

Рис. 15. Разбиение отрезка времени

Подсчитаем перемещение на каждом отрезке времени. Скорость прирастает на каждом интервале на:

На каждом отрезке мы будем считать движение равномерным и скорость приближенно равной начальной скорости на данном отрезке времени. Посмотрим, не приведет ли к ошибке наше приближение, если на небольшом промежутке движение будем считать равномерным. Максимальная ошибка будет равна:

и суммарная ошибка за всё время пути -> . При больших N принимаем ошибка близка к нулю. Это мы увидим и на графике (см. рис. 16): на каждом интервале будет ошибка, но суммарная ошибка при достаточно большом количестве интервалов будет пренебрежимо мала.

Рис. 16. Ошибка на интервалах

Итак, каждое следующее значение скорости на одну и ту же величину больше предыдущего. Из алгебры мы знаем, что это арифметическая прогрессия с разностью прогрессии :

Путь на участках (при равномерном прямолинейном движении (см. рис. 17) равен:


Рис. 17. Рассмотрение участков движения тела

На втором участке:

На n-м участке путь равен:

Арифметическая прогрессия

Арифметической прогрессией называется такая числовая последовательность, в которой каждое следующее число отличается от предыдущего на одну и ту же величину. Арифметическая прогрессия задается двумя параметрами: начальный член прогрессии и разность прогрессии . Тогда последовательность записывается так:

Сумма первых членов арифметической прогрессии вычисляется по формуле:

Просуммируем все пути. Это будет сумма первых N членов арифметической прогрессии:

Т. к. мы разбили движение на много интервалов, то можно считать, что , тогда:

У нас было множество формул, и, чтобы не запутаться, мы не писали каждый раз индексы х, но рассматривали всё в проекции на координатную ось.

Итак, мы получили главную формулу равноускоренного движения: перемещение при равноускоренном движении за время T, которую мы наряду с определением ускорения (изменение скорости за единицу времени) будем использовать для решения задач:

Мы занимались решением задачи об автомобиле. Подставим в решение числа и получим ответ: автомобиль проехал 55,4 км.

Математическая часть решения задачи

С перемещением мы разобрались. А как определить координату тела в любой момент времени?

По определению перемещение тела за время – это вектор, начало которого находится в начальной точке движения, а конец – в конечной точке, в которой тело будет через время . Нам нужно найти координату тела, поэтому запишем выражение для проекции перемещения на ось координат (см. рис. 18):

Рис. 18. Проекция перемещения

Выразим координату :

То есть координата тела в момент времени равна начальной координате плюс проекция перемещения, которое совершило тело за время . Проекцию перемещения при равноускоренном движении мы уже нашли, осталось подставить и записать:

Это и есть уравнение движения с постоянным ускорением. Оно позволяет узнать координату движущейся материальной точки в любой момент времени. Понятно, что момент времени мы выбираем в пределах промежутка, когда работает модель: ускорение постоянное, движение прямолинейное.

Почему уравнение движения нельзя применять для нахождения пути

В каких случаях мы можем считать перемещение по модулю равным пути? Когда тело движется вдоль прямой и не меняет направления. Например, при равномерном прямолинейном движении мы не всегда четко оговариваем, путь мы находим или перемещение, всё равно они совпадают.

При равноускоренном движении скорость изменяется. Если скорость и ускорение направлены в противоположные стороны (см. рис. 19), то модуль скорости убывает, и в какой-то момент он станет равен нулю и скорость поменяет направление, то есть тело начнет двигаться в противоположную сторону.

Рис. 19. Модуль скорости убывает

И тогда, если в данный момент времени тело находится на расстоянии 3 м от начала наблюдения, то его перемещение равно 3 м, но если тело сначала прошло 5 м, затем развернулось и прошло еще 2 м, то путь будет равен 7 м. И как же его найти, если не знать этих чисел? Просто надо найти момент, когда скорость равна нулю, то есть когда тело развернется, и найти путь к этой точке и от нее (см. рис. 20).

Рис. 20. Момент, когда скорость равна 0

Список литературы

  1. Соколович Ю.А., Богданова Г.С Физика: Справочник с примерами решения задач. – 2-е издание передел. – X.: Веста: Издательство «Ранок», 2005. – 464 с.
  2. Ландсберг Г.С. Элементарный учебник физики; т.1. Механика. Теплота. Молекулярная физика – М.: Издательство «Наука», 1985.
  1. Интернет портал «kaf-fiz-1586.narod.ru» ()
  2. Интернет портал «Учеба – Легко» ()
  3. Интернет портал «Гипермаркет знаний» ()

Домашнее задание

  1. Что такое арифметическая прогрессия?
  2. Какое движение называется поступательным?
  3. Чем характеризуется векторная величина?
  4. Запишите формулу для ускорения через изменение скорости.
  5. Какой вид имеет уравнение движения с постоянным ускорением?
  6. Вектор ускорения направлен в сторону движения тела. Как будет изменять свою скорость тело?

«Класс!ная физика» переезжает с “народа”!
«Класс!ная физика» – это сайт для тех, кто любит физику, учится сам и учит других.
«Класс!ная физика» – всегда рядом!
Интересные материалы по физике для школьников, учителей и всех любознательных.

Исходный сайт “Класс!ная физика” (class-fizika.narod.ru) с 2006 года входит в выпуски каталога «Образовательные ресурсы сети-интернет для основного общего и среднего (полного) общего образования», одобрено Министерством образования и науки РФ, Москва.

Читай, познавай, исследуй!
Мир физики интересен и увлекателен, он приглашает всех любознательных в путешествие по страницам сайта «Класс!ная физика».

А для начала – наглядная карта физики, которая показывает, откуда берут начало и как связаны между собой различные области физики, что они изучают, и для чего они нужны.
Карта Физики создана по видеоролику The Map of Physics от Доминика Вилиммана канала Domain of Science.


Физика и секреты художников

Тайны мумий фараонов и изобретения Ребрандта, подделки шедевров и секреты папирусов Древнего Египта – искусство скрывает в себе много тайн, но современные физики с помощью новых методов и приборов находят объяснения все большему числу удивительных секретов прошлого……… читать

Азбука физики

Всемогущее трение

Оно – всюду, да куда без него и денешься?
А вот три помощника-богатыря: графит, молебденит и тефлон. Эти удивительные вещества, обладающие очень высокой подвижностью частиц, применяются в настоящее время в качестве великолепной твердой смазки……… читать


Воздухоплавание

“Так поднимаются к звездам!” – начертано на гербе основателей воздухоплавания братьев Монгольфье.
Известный писатель Жюль Верн летал на воздушном шаре всего лишь 24 минуты, но это помогло ему создать увлекательнейшие художественные произведения……… читать


Паровые двигатели

“Этот могучий исполин был трёхметрового роста: гигант с лёгкостью тянул фургон с пятерыми пассажирами. На голове у Парового Человека была труба дымохода, откуда валил густой чёрный дым… всё, даже лицо, было сделано из железа, и все это непрерывно скрежетало и грохотало…” О ком это? Кому эти дифирамбы? ……… читать


Тайны магнита

Фалес Милетский наделял его душой, Платон сравнивал его с поэтом, Орфей находил его подобным жениху… В эпоху Возрождения магнит считали отображением неба и приписывали ему способность искривлять пространство. Японцы считали, что магнит – это сила, которая поможет повернуть к вам фортуну……… читать


По ту сторону зеркала

Знаете ли Вы, сколько интересных открытий может подарить “зазеркалье”? У изображения Вашего лица в зеркале правая и левая половины переставлены местами. А ведь лица редко бывают полностью симметричными, поэтому окружающие видят Вас совершенно иным. Задумывались ли Вы над этим? ……… читать


Секреты обыкновенного волчка

“Сознание того, что чудесное было рядом с нами, приходит слишком поздно.” – А.Блок.
Знаете ли Вы, что малайцы могут часами завороженно наблюдать за вращением волчка. Однако, требуется немалое умение, чтобы правильно раскрутить его, ведь вес малайского волчка может достигать нескольких килограммов……… читать


Изобретения Леонардо да Винчи

” Я хочу создавать чудеса!”-говорил он и спрашивал себя: “Но скажи мне, сделано ли тобою хоть что-нибудь?” Леонардо да Винчи писал свои трактаты тайнописью с помощью обыкновенного зеркала, поэтому его зашифрованные рукописи впервые смогли прочитать лишь три столетия спустя………

Движение с постоянным ускорением–это такое движение, при котором вектор ускорения остается постоянным как по величине, так и по направлению. Примером такого типа движения может служить движения точки в поле силы тяжести (как вертикально, так и под углом к горизонту).

Используя определение ускорения получим следующее соотношение

После интегрирования имеем равенство
.

С учетом того, что вектор мгновенной скорости есть
, будем иметь следующее выражение

Интегрирование последнего выражение дает следующее соотношение

. Откуда имеем получаем уравнение движения точки с постоянным ускорением


.

Примеры векторных уравнений движения материальной точки

Равномерное прямолинейное движение (
):

. (1.7)

Движение с постоянным ускорением (
):

. (1.8)

Зависимость скорости от времени при движении точки с постоянным ускорением имеет вид:

. (1.9)

Вопросы для самоконтроля.

    Сформулируйте определение механического движения.

    Дайте определение материальной точки.

    Каким образом определяется положение материальной точки в пространстве в векторном способе описания движения?

    В чем сущность векторного метода описания механического движения? Какие характеристики используются для описания этого движения?

    Дайте определения векторов средней и мгновенной скорости. Как определяется направление этих векторов?

    Дайте определение векторов среднего и мгновенного ускорений.

    Какое из соотношений является уравнением движения точки с постоянным ускорением? Какое соотношение определяет зависимость вектора скорости от времени?

§1.2. Координатный способ описания движения

В координатном способе для описания движения выбирают систему координат (например, декартову). Начало отсчета жестко закрепляют с выбранным телом (телом отсчета ). Пусть
единичные орты, направленные в положительные стороны осейOX, OY и OZ соответственно. Положение точки задается координатами
.

Вектор мгновенной скорости определяется следующим образом:

где
проекции вектора скорости на оси координат, а
производные от координат по времени.

Длина вектора скорости связана с его проекциями соотношением:

. (1.11)

Для вектора мгновенного ускорения справедливо соотношение:

где
проекции вектора ускорения на оси координат, а
производные по времени от проекций вектора скорости.

Длина вектора мгновенного ускорения находится по формуле:

. (1.13)

Примеры уравнений движения точки в декартовой системе координат


. (1.14)


Уравнения движения:
. (1.15)

Зависимости проекций вектора скорости на оси координат от времени:

(1.16)

Вопросы для самоконтроля.

    В чем сущность координатного способа описания движения?

    Каким соотношением определяется вектор мгновенной скорости? По какой формуле вычисляется величина вектора скорости?

    Каким соотношением определяется вектор мгновенного ускорения? По какой формуле вычисляется величина вектора мгновенного ускорения?

    Какие соотношения называют уравнениями равномерного движения точки?

    Какие соотношения называют уравнениями движения с постоянным ускорением? По каким формулам рассчитывают проекции мгновенной скорости точки на оси координат?

Изучением классического механического движения в физике занимается кинематика. В отличие от динамики, наука изучает, почему движутся тела. Она отвечает на вопрос, как они это делают. В данной статье рассмотрим, что такое ускорение и движение с постоянным ускорением.

Понятие об ускорении

Когда тело движется в пространстве, за некоторое время оно преодолевает определенный путь, который является длиной траектории. Чтобы рассчитать этот путь, пользуются понятиями скорости и ускорения.

Скорость как физическая величина характеризует быстроту во времени изменения пройденного пути. Скорость направлена по касательной к траектории в сторону перемещения тела.

Ускорение — это несколько более сложная величина. Говоря кратко, она описывает изменение скорости в рассматриваемый момент времени. Математическое выглядит так:

Чтобы яснее понять эту формулу, приведем простой пример: предположим, что за 1 секунду движения скорость тела увеличилась на 1 м/с. Эти цифры, подставленные в выражение выше, приводят к результату: ускорение тела в течение этой секунды было равно 1 м/с 2 .

Направление ускорения совершенно не зависит от направления скорости. Его вектор совпадает с вектором результирующей силы, которая вызывает это ускорение.

Следует отметить важный момент в приведенном определении ускорения. Эта величина характеризует не только изменение скорости по модулю, но и по направлению. Последний факт следует учитывать в случае криволинейного движения. Далее в статье будет рассматриваться только прямолинейное движение.

Скорость при движении с постоянным ускорением

Ускорение является постоянным, если оно в процессе движения сохраняет свой модуль и направление. Такое движение называют равноускоренным или равнозамедленным — все зависит от того, приводит ли ускорение к увеличению скорости или к ее уменьшению.

В случае движения тела с постоянным ускорением определить скорость можно по одной из следующих формул:

Первые два уравнения характеризуют равноускоренное перемещение. Отличие между ними заключается в том, что второе выражение применимо для случая ненулевой начальной скорости.

Третье уравнение — это выражение для скорости при равнозамедленном движении с постоянным ускорением. Ускорение при этом направлено против скорости.

Графиками всех трех функций v(t) являются прямые. В первых двух случаях прямые имеют положительный наклон относительно оси абсцисс, в третьем случае этот наклон является отрицательным.

Формулы пройденного пути

Для пути в случае движения с ускорением постоянным (ускорение a = const) получить формулы несложно, если вычислить интеграл от скорости по времени. Проделав эту математическую операцию для записанных выше трех уравнений, мы получим следующие выражения для пути L:

L = v 0 *t + a*t 2 /2;

L = v 0 *t – a*t 2 /2.

Графиками всех трех функций пути от времени являются параболы. В первых двух случаях правая ветвь параболы возрастает, а для третьей функции она постепенно выходит на некоторую константу, которая соответствует пройденному пути до полной остановки тела.

Решение задачи

Двигаясь со скоростью 30 км/ч, автомобиль начал ускоряться. За 30 секунд он прошел расстояние 600 метров. Чему было равно ускорение автомобиля?

В первую очередь переведем начальную скорость из км/ч в м/с:

v 0 = 30 км/ч = 30000/3600 = 8,333 м/с.

Теперь запишем уравнение движения:

L = v 0 *t + a*t 2 /2.

Из этого равенства выразим ускорение, получим:

a = 2*(L – v 0 *t)/t 2 .

Все физические величины в этом уравнении известны из условия задачи. Подставляем их в формулу и получаем ответ: a ≈ 0,78 м/с 2 . Таким образом, двигаясь с ускорением постоянным, автомобиль за каждую секунду увеличивал свою скорость на 0,78 м/с.

Рассчитаем также (для интереса), какую скорость он приобрел через 30 секунд ускоренного движения, получаем:

v = v 0 + a*t = 8,333 + 0,78*30 = 31,733 м/с.

Полученная скорость равна 114,2 км/ч.

Цели урока:

Образовательные:

Развивающие:

Воспитательные

Тип урока : Комбинированный урок.

Просмотр содержимого документа


«Тема урока: «Ускорение. Прямолинейное движение с постоянным ускорением».»

Подготовила – учитель физики МБОУ «СОШ №4» Погребняк Марина Николаевна

Класс -11

Урок 5/4 Тема урока: «Ускорение. Прямолинейное движение с постоянным ускорением ».

Цели урока:

Образовательные: Познакомить учащихся с характерными особенностями прямолинейного равноускоренного движения. Дать понятие об ускорении как основной физической величине, характеризующей неравномерное движение. Вввести формулу для определения мгновенной скорости тела в любой момент времени, рассчитывать мгновенную скорость тела в любой момент времени,

совершенствовать умения учащихся решать задачи аналитическим и графическим способами.

Развивающие: развитие у школьников теоретического, творческого мышления, формирование операционного мышления, направленного на выбор оптимальных решений

Вос питательные : воспитывать сознательное отношение к учебе и заинтересованность в изучении физики.

Тип урока : Комбинированный урок.

Демонстрации:

1. Равноускоренное движение шарика по наклонной плоскости.

2. Мультимедийное приложение «Основы кинематики»: фрагмент «Равноускоренное движение».

Ход работы.

1.Организационный момент .

2. Проверка знаний : Самостоятельная работа («Перемещение.» «Графики прямолинейного равномерного движения») – 12 мин.

3. Изучение нового материала.

План изложения нового материала:

1. Мгновенная скорость.

2. Ускорение.

3. Скорость при прямолинейном равноускоренном движении.

1. Мгновенная скорость. Если скорость тела изменяется со временем, для описания движения надо знать, чему равна скорость тела в данный момент времени (или в данной точке траектории). Эта скорость называется мгновенной скоростью.

Можно также сказать, что мгновенная скорость – это средняя скорость за очень малый интервал времени. При движении с переменной скоростью средняя скорость, измеренная за различные интервалы времени, будет разной.

Однако, если при измерении средней скорости брать все меньшие и меньшие интервалы времени, значение средней скорости будет стремиться к некоторому определенному значению. Это и есть мгновенная скорость в данный момент времени. В дальнейшем, говоря о скорости тела, мы будем иметь в виду его мгновенную скорость.

2. Ускорение. При неравномерном движении мгновенная скорость тела – величина переменная; она различна по модулю и (или) по направлению в разные моменты времени и в разных точках траектории. Все спидометры автомобилей и мотоциклов показывают нам только модуль мгновенной скорости.

Если мгновенная скорость неравномерного движения изменяется неодинаково за одинаковые промежутки времени, то рассчитать ее очень трудно.

Такие сложные неравномерные движения в школе не изучаются. Поэтому рассмотрим только самое простое неравномерное движение – равноускоренное прямолинейное.

Прямолинейное движение, при котором мгновенная скорость за любые равные интервалы времени изменяется одинаково, называют равноускоренным прямолинейным движением.

Если скорость тела при движении изменяется, возникает вопрос: какова «скорость изменения скорости»? Эта величина, называемая ускорением, играет важнейшую роль во всей механике: вскоре мы увидим, что ускорение тела определяется действующими на это тело силами.

Ускорением называется отношение изменения скорости тела к интервалу времени, за который это изменение произошло.

Единица измерения ускорения в СИ: м/с 2 .

Если тело движется в одном направлении с ускорением 1 м/с 2 , его скорость изменяется каждую секунду на 1 м/с.

Термин «ускорение» используется в физике, когда речь идет о любом изменении скорости, в том числе и тогда, когда модуль скорости уменьшается или когда модуль скорости остается неизменным и скорость изменяется только по направлению.

3. Скорость при прямолинейном равноускоренном движении.

Из определения ускорения следует, что v = v 0 + at.

Если направить ось х вдоль прямой, по которой движется тело, то в проекциях на ось х получим v x = v 0 x + a x t.

Таким образом, при прямолинейном равноускоренном движении проекция скорости линейно зависит от времени. Это означает, что графиком зависимости v x (t) является отрезок прямой.

Формула перемещения:

График скорости разгоняющегося автомобиля:

График скорости тормозящего автомобиля

4. Закрепление нового материала.

Чему равна мгновенная скорость камня, брошенного вертикально вверх, в верхней точке траектории?

О какой скорости – средней или мгновенной – идет речь в следующих случаях:

а) поезд прошел путь между станциями со скоростью 70 км/ч;

б) скорость движения молотка при ударе равна 5 м/с;

в) скоростемер на электровозе показывает 60 км/ч;

г) пуля вылетает из винтовки со скоростью 600 м/с.

ЗАДАЧИ, РЕШАЕМЫЕ НА УРОКЕ

Ось ОХ направлена вдоль траектории прямолинейного движения тела. Что вы можете сказать о движении, при котором: a) v x 0, а х 0; б) v x 0, а х v x х 0;

г) v x х v x х = 0?

1. Хоккеист слегка ударил клюшкой по шайбе, придав ей скорость 2 м/с. Чему будет равна скорость шайбы через 4 с после удара, если в результате трения о лед она движется с ускорением 0,25 м/с 2 ?

2. Поезд через 10 с после начала движения приобретает скорость 0,6 м/с. Через сколько времени от начала движения скорость поезда станет равна 3м/с?

5.ДОМАШНЕЕ ЗАДАНИЕ : §5,6, упр. 5 №2, упр. 6 №2.

Ускорение движение с постоянным ускорением единица ускорения. §1.20

Ускорение. Прямолинейное движение с постоянным ускорением. Мгновеннная скорость.

Ускорение показывает, как быстро меняется скорость тела.

t 0 = 0c v 0 = 0 м/с Скорость изменилась на v = v 2 – v 1 в течение

t 1 = 5c v 1 = 2 м/ с промежутка времени = t 2 – t 1 . Значит за 1 с скорость

t 2 = 10c v 2 = 4 м/с тела увеличится на = .

t 3 = 15c v 3 = 6 м/с = или = . (1 м/с 2)

Ускорение – векторная величина, равная отношению изменения скорости к промежутку времени, в течение которого это изменение произошло.

Физический смысл : а = 3 м/с 2 – это значит, что за 1 с модуль скорости меняется на 3 м/с.

Если тело разгоняется а>0, если тормозит а


Аt = ; = + аt мгновенная скорость тела в любой момент времени. (Функция v(t)).

Перемещение при равноускоренном движении. Уравнение движения

Для равномерного движения S=v*t, где v и t являются сторонами прямоугольника под графиком скорости. Т.е. перемещение = площади фигуры под графиком скорости.


Аналогично можно найти перемещение при равноускоренном движении. Нужно всего лишь найти отдельно площадь прямоугольника, треугольника и сложить их. Площадь прямоугольника v 0 t, площадь треугольника (v-v 0)t/2, где мы делаем замену v – v 0 = аt . Получим s = v 0 t + аt 2 /2

s = v 0 t + аt 2 /2

Формула перемещения при равноускоренном движении

Учитывая, что вектор s = х-х 0 , получим х-х 0 = v 0 t + аt 2 /2 или вынесем начальную координату вправо х = х 0 + v 0 t + аt 2 /2

х = х 0 + v 0 t + аt 2 /2

По этой формуле можно найти координату ускоренно движущегося тела в любой момент времени

При равнозамедленном движении перед буквой «а» в формулах знак + можно заменить на –

Движение. Теплота Китайгородский Александр Исаакович

Прямолинейное движение с постоянным ускорением

Такое движение возникает, согласно закону Ньютона, тогда, когда в сумме на тело действует постоянная сила, подгоняющая или тормозящая тело.

Хотя и не вполне точно, такие условия возникают довольно часто: тормозится под действием примерно постоянной силы трения автомашина, идущая с выключенным мотором, падает с высоты под действием постоянной силы тяжести увесистый предмет.

Зная величину результирующей силы, а также массу тела, мы найдем по формуле a = F /m величину ускорения. Так как

где t – время движения, v – конечная, а v 0 – начальная скорость, то при помощи этой формулы можно ответить на ряд вопросов такого, например, характера: через сколько времени остановится поезд, если известна сила торможения, масса поезда и начальная скорость? До какой скорости разгонится автомашина, если известна сила мотора, сила сопротивления, масса машины и время разгона?

Часто нам бывает интересно знать длину пути, пройденного телом в равномерно-ускоренном движении. Если движение равномерное, то пройденный путь находится умножением скорости движения на время движения. Если движение равномерно-ускоренное, то подсчет величины пройденного пути производится так, как если бы тело двигалось то же время t равномерно со скоростью, равной полусумме начальной и конечной скоростей:

Итак, при равномерно-ускоренном (или замедленном) движении путь, пройденный телом, равен произведению полусуммы начальной и конечной скоростей на время движения. Такой же путь был бы пройден за то же время при равномерном движении со скоростью (1/2)(v 0 + v ). В этом смысле про (1/2)(v 0 + v ) можно сказать, что это средняя скорость равномерно-ускоренного движения.

Полезно составить формулу, которая показывала бы зависимость пройденного пути от ускорения. Подставляя v = v 0 + at в последнюю формулу, находим:

или, если движение происходит без начальной скорости,

Если за одну секунду тело прошло 5 м, то за две секунды оно пройдет (4?5) м, за три секунды – (9?5) м и т.д. Пройденный путь возрастает пропорционально квадрату времени.

По этому закону падает с высоты тяжелое тело. Ускорение при свободном падении равно g , и формула приобретает такой вид:

если t подставить в секундах.

Если бы тело могло падать без помех каких-нибудь 100 секунд, то оно прошло бы с начала падения громадный путь – около 50 км. При этом за первые 10 секунд будет пройдено всего лишь (1/2) км – вот что значит ускоренное движение.

Но какую же скорость разовьет тело при падении с заданной высоты? Для ответа на этот вопрос нам понадобятся формулы, связывающие пройденный путь с ускорением и скоростью. Подставляя в S = (1/2)(v 0 + v )t значение времени движения t = (v ? v 0)/a , получим:

или, если начальная скорость равна нулю,

Десять метров – это высота небольшого двух- или трехэтажного дома. Почему опасно прыгнуть на Землю с крыши такого дома? Простой расчет показывает, что скорость свободного падения достигнет значения v = sqrt(2·9,8·10) м/с = 14 м/с? 50 км/ч, а ведь это городская скорость автомашины.

Сопротивление воздуха не намного уменьшит эту скорость.

Выведенные нами формулы применяются для самых различных расчетов. Применим их, чтобы посмотреть, как происходит движение на Луне.

В романе Уэллса «Первые люди на Луне» рассказывается о неожиданностях, испытанных путешественниками в их фантастических прогулках. На Луне ускорение тяжести примерно в 6 раз меньше земного. Если на Земле падающее тело проходит за первую секунду 5 м, то на Луне оно «проплывет» вниз всего лишь 80 см (ускорение равно примерно 1,6 м/с 2).

Прыжок с высоты h длится время t = sqrt(2h /g ). Так как лунное ускорение в 6 раз меньше земного, то на Луне для прыжка понадобится в sqrt(6) ? 2,45 раз больше времени. Во сколько же раз уменьшается конечная скорость прыжка (v = sqrt(2gh ))?

На Луне можно безопасно прыгнуть с крыши трехэтажного дома. В шесть раз возрастает высота прыжка, cделанного с той же начальной скоростью (формула h = v 2 /(2g )). Прыжок, превышающий земной рекорд, будет под силу ребенку.

Из книги Физика: Парадоксальная механика в вопросах и ответах автора Гулиа Нурбей Владимирович

4. Движение и сила

Из книги Новейшая книга фактов. Том 3 [Физика, химия и техника. История и археология. Разное] автора Кондрашов Анатолий Павлович

Из книги Теория Вселенной автора Этэрнус

Из книги Занимательно об астрономии автора Томилин Анатолий Николаевич

9. Движение Луны Луна обращается вокруг Земли с периодом в 27 дней 7 часов 43 минуты и 11,5 секунды. Этот период называется звездным, или сидерическим, месяцем. Точно с таким же периодом обращается Луна и вокруг собственной оси. Поэтому понятно, что к нам постоянно обращена

Из книги Эволюция физики автора Эйнштейн Альберт

Эфир и движение Принцип относительности Галилея справедлив для механических явлений. Во всех инерциальных системах, движущихся относительно друг друга, применимы одни и те же законы механики. Справедлив ли этот принцип и для немеханических явлений, особенно тех, для

Из книги Физика на каждом шагу автора Перельман Яков Исидорович

Движение по кругу Раскройте зонтик, уприте его концом в пол, закружите и бросьте внутрь мячик, скомканную бумагу, носовой платок – вообще что-нибудь легкое и неломкое. Произойдет нечто для вас неожиданное. Зонтик словно не пожелает принять подарка: мяч или бумажный ком

Из книги Движение. Теплота автора Китайгородский Александр Исаакович

Движение относительно Закон инерции приводит нас к выводу о множественности инерциальных систем.Не одна, а множество систем отсчета исключают «беспричинные» движения.Если одна такая система найдена, то сразу же найдется и другая, движущаяся поступательно (без

Из книги Системы мира (от древних до Ньютона) автора Гурев Григорий Абрамович

Движение по окружности Если точка движется по окружности, то движение является ускоренным, уже хотя бы потому, что в каждый момент времени скорость меняет свое направление. По величине скорость может оставаться неизменной, и мы остановим внимание именно на подобном

Из книги 1. Современная наука о природе, законы механики автора Фейнман Ричард Филлипс

Реактивное движение Человек движется, отталкиваясь от земли; лодка плывет потому, что гребцы отталкиваются веслами от воды; теплоход также отталкивается от воды, только не веслами, а винтами. Также отталкиваются от земли и поезд, идущий по рельсам, и автомашина, –

Из книги Фарадей. Электромагнитная индукция [Наука высокого напряжения] автора Кастильо Сержио Рарра

VI. Движение твердых тел Момент силы Попробуйте рукой привести во вращение тяжелое маховое колесо. Тяните за спицу. Вам будет тяжело, если вы ухватитесь рукой слишком близко к оси. Переместите руку к ободу, и дело пойдет легче.Что же изменилось? Ведь сила в обоих случаях

Из книги автора

Как выглядит тепловое движение Взаимодействие между молекулами может иметь большее или меньшее значение в «жизни» молекул.Три состояния вещества – газообразное, жидкое и твердое – различаются одно от другого той ролью, которую в них играет взаимодействие

Из книги автора

ПРЕВРАТИТЬ ЭЛЕКТРИЧЕСТВО В ДВИЖЕНИЕ Фарадей заметил в опытах Эрстеда одну маленькую деталь, которая, как казалось, содержала ключ к пониманию проблемы.Он догадался, что магнетизм электрического тока всегда отклоняет стрелку компаса в одну сторону. Например, если

План-конспект урока по теме «Скорость при прямолинейном движении с постоянным ускорением»

Дата :

Тема: «Скорость при прямолинейном движении с постоянным ускорением»

Цели:

Образовательная

: Обеспечить и сформировать осознанное усвоение знаний о скорости при прямолинейном движении с постоянным ускорением;

Развивающая

: Продолжить развитие навыков самостоятельной деятельности, навыков работы в группах.

Воспитательная

: Формировать познавательный интерес к новым знаниям; воспитывать дисциплину поведения.

Тип урока: урок усвоения новых знаний

Оборудование и источники информации:

    Исаченкова, Л. А. Физика: учеб. для 9 кл. учреждений общ. сред. образования с рус. яз. обучения / Л. А. Исаченкова, Г. В. Пальчик, А. А. Сокольский; под ред. А. А. Сокольского. Минск: Народная асвета, 2015

    Исаченкова, Л. А. Сборник задач по физике. 9 класс: пособие для учащихся учреждений общ. сред. образования с рус. яз. обучения / Л. А. Исаченкова, Г. В. Пальчик, В. В. Дорофейчик. Минск: Аверсэв, 2016, 2017.

Структура урока:

    Организационный момент(5 мин)

    Актуализация опорных знаний(5мин)

    Изучение нового материала (15 мин)

    Физкультминутка (2 мин)

    Закрепление знаний (13мин)

    Итоги урока(5 мин)

    Организационный момент

Здравствуйте, садитесь! (Проверка присутствующих).

Сегодня на уроке мы должны разобраться со скоростью при прямолинейном движении с постоянным ускорением. А это значит, что Тема урока : Скорость при прямолинейном движении с постоянным ускорением

    Актуализация опорных знаний

Самое простое из всех неравномерных движении

прямолинейное движение с постоянным ускорением. Его называют равнопеременным.

Как изменяется скорость тела при равнопеременном движении?

    Изучение нового материала

Рассмотрим движение стального шарика по наклонному желобу. Опыт показывает, что его ускорение практически постоянно:

Пусть в момент времени t = 0 шарик имел начальную скорость (рис. 83).

Как найти зависимость скорости шарика от времени?

Ускорение шарика

а = . В нашем примере Δt = t , Δ – . Значит,

,

откуда

При движении с постоянным ускорением скорость тела линейно зависит от

времени.

Из равенств (1 ) и (2) следуют формулы для проекций:

Построим графики зависимости

a x ( t ) и v x ( t ) (рис. 84, а, б).

Рис. 84

Согласно рисунку 83

а х = а > 0, = v 0 > 0.

Тогда

зависимости a x ( t ) соответствует график 1 (см. рис. 84, а). Это прямая, параллельная оси времени. Зависимости v x ( t ) соответствует график , описывающий возрастание проекции ско рости (см. рис. 84, б). Понятно, что растет и модуль скорости. Шарик движется равноускоренно.

Рассмотрим второй пример (рис. 85). Теперь начальная скорость шарика направлена вдоль желоба вверх. Двигаясь вверх, шарик будет постепенно терять скорость. В точке

А он на мгновение остановится и начнет скатываться вниз. Точку A называют точкой поворота.

Согласно рисунку 85 а х = – а 0, = v 0 > 0, и формулам (3) и (4) соответствуют графики 2 и 2″ (см. рис. 84, а , б).

График 2″ показывает, что вначале, пока шарик двигался вверх, проекция скорости v x была положительна. Она уменьшалась и в момент времени t = стала равной нулю. В этот момент шарик достиг точки поворота A (см. рис. 85). В данной точке направление скорости шарика изменилось на противоположное и при t > проекция скорости стала отрицательной.

Из графика 2″ (см. рис. 84, б) видно также, что до момента поворота модуль скорости уменьшался – шарик двигался вверх равнозамедленно. При t > t n модуль скорости растет – шарик движется вниз равноускоренно.

Постройте самостоятельно графики зависимости модуля скорости от времени для обоих примеров.

Какие еще закономерности равнопеременного движения необходимо знать?

В § 8 мы доказали, что для равномерного прямолинейного движения площадь фигуры между графиком

v x и осью времени (см. рис. 57) численно равна проекции перемещения Δ r х . Можно доказать, что это правило применимо и для неравномерного движения. Тогда согласно рисунку 86 проекция перемещения Δ r х при равнопеременном движении определяется площадью трапеции ABCD . Эта площадь равна полусумме оснований трапеции умноженной на ее высоту AD .

В результате:

Так как среднее значение проекции скорости формулы (5)

следует:

При движении с постоянным ускорением соотношение (6) выполняется не только для проекции, но и для векторов скорости:

Средняя скорость движения с постоянным ускорением равна полусумме начальной и конечной скоростей.

Формулы (5), (6) и (7) нельзя использовать

для движения с непостоянным ускорением. Это может привести к грубым ошибкам.

    Закрепление знаний

Разберем пример решения задачи со страницы 57:

Автомобиль двигался со скоростью, модуль которой = 72

. Увидев красный свет светофора, водитель на участке пути s = 50 м равномерно снизил скорость до = 18 . Определите характер движения автомобиля. Найдите направление и модуль ускорения, с которым двигался автомобиль при торможении.

Дано: Реше

ние:

72 = 20 Движение автомобиля было равнозамедленным. Уско-

рение автомобиля

направлено противоположно

18 = 5 скорости его движения.

Модуль ускорения:

s

= 50 м

Время торможения:

а – ? Δ t =

Тогда

Ответ:

    Итоги урока

    При движении с постоянным ускорением скорость линейно зависит от времени.

    При равноускоренном движении направления мгновенной скорости и ускорения совпадают, при равнозамедленном – они противоположны.

    Средняя скорость движения

с постоянным ускорением равна полусумме начальной и конечной скоростей.

Организация домашнего задания

§ 12, упр. 7 № 1, 5

Рефлексия.

Продолжите фразы:

    Сегодня на уроке я узнал…

    Было интересно…

    Знания, которые я получил на уроке, пригодятся

Изучением классического механического движения в физике занимается кинематика. В отличие от динамики, наука изучает, почему движутся тела. Она отвечает на вопрос, как они это делают. В данной статье рассмотрим, что такое ускорение и движение с постоянным ускорением.

Понятие об ускорении

Когда тело движется в пространстве, за некоторое время оно преодолевает определенный путь, который является длиной траектории. Чтобы рассчитать этот путь, пользуются понятиями скорости и ускорения.

Скорость как физическая величина характеризует быстроту во времени изменения пройденного пути. Скорость направлена по касательной к траектории в сторону перемещения тела.

Ускорение — это несколько более сложная величина. Говоря кратко, она описывает изменение скорости в рассматриваемый момент времени. Математическое выглядит так:

Чтобы яснее понять эту формулу, приведем простой пример: предположим, что за 1 секунду движения скорость тела увеличилась на 1 м/с. Эти цифры, подставленные в выражение выше, приводят к результату: ускорение тела в течение этой секунды было равно 1 м/с 2 .

Направление ускорения совершенно не зависит от направления скорости. Его вектор совпадает с вектором результирующей силы, которая вызывает это ускорение.

Следует отметить важный момент в приведенном определении ускорения. Эта величина характеризует не только изменение скорости по модулю, но и по направлению. Последний факт следует учитывать в случае криволинейного движения. Далее в статье будет рассматриваться только прямолинейное движение.

Скорость при движении с постоянным ускорением

Ускорение является постоянным, если оно в процессе движения сохраняет свой модуль и направление. Такое движение называют равноускоренным или равнозамедленным — все зависит от того, приводит ли ускорение к увеличению скорости или к ее уменьшению.

В случае движения тела с постоянным ускорением определить скорость можно по одной из следующих формул:

Первые два уравнения характеризуют равноускоренное перемещение. Отличие между ними заключается в том, что второе выражение применимо для случая ненулевой начальной скорости.

Третье уравнение — это выражение для скорости при равнозамедленном движении с постоянным ускорением. Ускорение при этом направлено против скорости.

Графиками всех трех функций v(t) являются прямые. В первых двух случаях прямые имеют положительный наклон относительно оси абсцисс, в третьем случае этот наклон является отрицательным.

Формулы пройденного пути

Для пути в случае движения с ускорением постоянным (ускорение a = const) получить формулы несложно, если вычислить интеграл от скорости по времени. Проделав эту математическую операцию для записанных выше трех уравнений, мы получим следующие выражения для пути L:

L = v 0 *t + a*t 2 /2;

L = v 0 *t – a*t 2 /2.

Графиками всех трех функций пути от времени являются параболы. В первых двух случаях правая ветвь параболы возрастает, а для третьей функции она постепенно выходит на некоторую константу, которая соответствует пройденному пути до полной остановки тела.

Решение задачи

Двигаясь со скоростью 30 км/ч, автомобиль начал ускоряться. За 30 секунд он прошел расстояние 600 метров. Чему было равно ускорение автомобиля?

В первую очередь переведем начальную скорость из км/ч в м/с:

v 0 = 30 км/ч = 30000/3600 = 8,333 м/с.

Теперь запишем уравнение движения:

L = v 0 *t + a*t 2 /2.

Из этого равенства выразим ускорение, получим:

a = 2*(L – v 0 *t)/t 2 .

Все физические величины в этом уравнении известны из условия задачи. Подставляем их в формулу и получаем ответ: a ≈ 0,78 м/с 2 . Таким образом, двигаясь с ускорением постоянным, автомобиль за каждую секунду увеличивал свою скорость на 0,78 м/с.

Рассчитаем также (для интереса), какую скорость он приобрел через 30 секунд ускоренного движения, получаем:

v = v 0 + a*t = 8,333 + 0,78*30 = 31,733 м/с.

Полученная скорость равна 114,2 км/ч.

Уравнение движения с постоянным ускорением. Поступательное движение. Движение с постоянным ускорением

На данном уроке, тема которого: «Уравнение движения с постоянным ускорением. Поступательное движение», мы вспомним, что такое движение, каким оно бывает. Также вспомним, что такое ускорение, рассмотрим уравнение движения с постоянным ускорением и как им пользоваться для определения координаты движущегося тела. Рассмотрим пример задачи для закрепления материала.

Главная задача кинематики – определить положение тела в любой момент времени. Тело может покоиться, тогда его положение меняться не будет (см. рис. 1).

Рис. 1. Покоящееся тело

Тело может двигаться прямолинейно с постоянной скоростью. Тогда его перемещение будет изменяться равномерно, то есть одинаково за равные промежутки времени (см. рис. 2).

Рис. 2. Перемещение тела при движении с постоянной скоростью

Перемещение , скорость, умноженная на время, это мы давно умеем делать. Тело может двигаться с постоянным ускорением, рассмотрим такой случай (см. рис. 3).

Рис. 3. Движение тела с постоянным ускорением

Ускорение

Ускорение – это изменение скорости за единицу времени (см. рис. 4):

Рис. 4. Ускорение

Скорость – векторная величина, поэтому и изменение скорости, т. е. разность векторов конечной и начальной скорости, является вектором. Ускорение – тоже вектор, направленный туда же, куда и вектор разности скоростей (см. рис. 5).

Мы рассматриваем прямолинейное движение, поэтому можно выбрать координатную ось вдоль прямой, вдоль которой происходит движение, и рассматривать проекции векторов скорости и ускорения на эту ось:

Тогда равномерно изменяется его скорость: (если его начальная скорость была равна нулю). Как теперь найти перемещение? Скорость умножить на время – нельзя : скорость постоянно менялась; какую брать? Как определить, где при таком движении будет находиться тело в любой момент времени – сегодня мы эту проблему решим.

Сразу определимся с моделью: мы рассматриваем прямолинейное поступательное движение тела. В таком случае можем применять модель материальной точки. Ускорение направлено вдоль той же прямой, вдоль которой материальная точка движется (см. рис. 6).

Поступательное движение

Поступательное движение – это такое движение, при котором все точки тела движутся одинаково: с одинаковой скоростью, совершая одинаковое перемещение (см. рис. 7).

Рис. 7. Поступательное движение

А как еще может быть? Взмахните рукой и проследите: понятно, что ладонь и плечо двигались по-разному. Посмотрите на колесо обозрения: точки вблизи оси почти не движутся, а кабинки движутся с другой скоростью и по другим траекториям (см. рис. 8).

Рис. 8. Движение выбранных точек на колесе обозрения

Посмотрите на движущийся автомобиль: если не учитывать вращение колес и движение частей мотора, все точки автомобиля движутся одинаково, движение автомобиля считаем поступательным (см. рис. 9).

Рис. 9. Движение автомобиля

Тогда нет смысла описывать движение каждой точки, можно описать движение одной. Автомобиль считаем материальной точкой. Обратите внимание, что при поступательном движении линия, соединяющая любые две точки тела при движении, остается параллельной сама себе (см. рис. 10).

Рис. 10. Положение линии, соединяющей две точки

Автомобиль ехал прямолинейно в течение часа. В начале часа его скорость была 10 км/ч, а в конце – 100 км/ч (см. рис. 11).

Рис. 11. Рисунок к задаче

Скорость изменялась равномерно. Сколько километров проехал автомобиль?

Проанализируем условие задачи.

Скорость автомобиля изменялась равномерно, то есть всё время пути его ускорение было постоянным. Ускорение по определению равно:

Автомобиль ехал прямолинейно, поэтому мы можем рассматривать его движение в проекции на одну ось координат:

Найдем перемещение.

Пример возрастающей скорости

На стол кладут орехи, по одному ореху в минуту. Понятно: сколько минут пройдет, столько орехов на столе окажется. А теперь представим, что скорость накладывания орехов равномерно возрастает с нуля: первую минуту орехов не кладут, во вторую кладут один орех, потом два, три и так далее. Сколько орехов окажется на столе через какое-то время? Понятно, что меньше, чем если бы максимальная скорость поддерживалась всегда. Причем хорошо видно, что меньше в 2 раза (см. рис. 12).

Рис. 12. Количество орехов при разной их скорости выкладывании

Так же и с равноускоренным движением: допустим, сначала скорость была равна нулю, в конце стала равна (см. рис. 13).

Рис. 13. Изменение скорости

Если бы тело постоянно двигалось с такой скоростью, его перемещение было бы равно , но поскольку скорость равномерно возрастала – то в 2 раза меньше.

Мы умеем находить перемещение при РАВНОМЕРНОМ движении: . Как обойти эту проблему? Если скорость изменяется не на много, то движение можно приближенно считать равномерным. Изменение скорости будет небольшим за небольшой интервал времени (см. рис. 14).

Рис. 14. Изменение скорости

Поэтому разобьем время в пути T на N небольших отрезков длительностью (см. рис. 15).

Рис. 15. Разбиение отрезка времени

Подсчитаем перемещение на каждом отрезке времени. Скорость прирастает на каждом интервале на:

На каждом отрезке мы будем считать движение равномерным и скорость приближенно равной начальной скорости на данном отрезке времени. Посмотрим, не приведет ли к ошибке наше приближение, если на небольшом промежутке движение будем считать равномерным. Максимальная ошибка будет равна:

и суммарная ошибка за всё время пути -> . При больших N принимаем ошибка близка к нулю. Это мы увидим и на графике (см. рис. 16): на каждом интервале будет ошибка, но суммарная ошибка при достаточно большом количестве интервалов будет пренебрежимо мала.

Рис. 16. Ошибка на интервалах

Итак, каждое следующее значение скорости на одну и ту же величину больше предыдущего. Из алгебры мы знаем, что это арифметическая прогрессия с разностью прогрессии :

Путь на участках (при равномерном прямолинейном движении (см. рис. 17) равен:


Рис. 17. Рассмотрение участков движения тела

На втором участке:

На n-м участке путь равен:

Арифметическая прогрессия

Арифметической прогрессией называется такая числовая последовательность, в которой каждое следующее число отличается от предыдущего на одну и ту же величину. Арифметическая прогрессия задается двумя параметрами: начальный член прогрессии и разность прогрессии . Тогда последовательность записывается так:

Сумма первых членов арифметической прогрессии вычисляется по формуле:

Просуммируем все пути. Это будет сумма первых N членов арифметической прогрессии:

Т. к. мы разбили движение на много интервалов, то можно считать, что , тогда:

У нас было множество формул, и, чтобы не запутаться, мы не писали каждый раз индексы х, но рассматривали всё в проекции на координатную ось.

Итак, мы получили главную формулу равноускоренного движения: перемещение при равноускоренном движении за время T, которую мы наряду с определением ускорения (изменение скорости за единицу времени) будем использовать для решения задач:

Мы занимались решением задачи об автомобиле. Подставим в решение числа и получим ответ: автомобиль проехал 55,4 км.

Математическая часть решения задачи

С перемещением мы разобрались. А как определить координату тела в любой момент времени?

По определению перемещение тела за время – это вектор, начало которого находится в начальной точке движения, а конец – в конечной точке, в которой тело будет через время . Нам нужно найти координату тела, поэтому запишем выражение для проекции перемещения на ось координат (см. рис. 18):

Рис. 18. Проекция перемещения

Выразим координату :

То есть координата тела в момент времени равна начальной координате плюс проекция перемещения, которое совершило тело за время . Проекцию перемещения при равноускоренном движении мы уже нашли, осталось подставить и записать:

Это и есть уравнение движения с постоянным ускорением. Оно позволяет узнать координату движущейся материальной точки в любой момент времени. Понятно, что момент времени мы выбираем в пределах промежутка, когда работает модель: ускорение постоянное, движение прямолинейное.

Почему уравнение движения нельзя применять для нахождения пути

В каких случаях мы можем считать перемещение по модулю равным пути? Когда тело движется вдоль прямой и не меняет направления. Например, при равномерном прямолинейном движении мы не всегда четко оговариваем, путь мы находим или перемещение, всё равно они совпадают.

При равноускоренном движении скорость изменяется. Если скорость и ускорение направлены в противоположные стороны (см. рис. 19), то модуль скорости убывает, и в какой-то момент он станет равен нулю и скорость поменяет направление, то есть тело начнет двигаться в противоположную сторону.

Рис. 19. Модуль скорости убывает

И тогда, если в данный момент времени тело находится на расстоянии 3 м от начала наблюдения, то его перемещение равно 3 м, но если тело сначала прошло 5 м, затем развернулось и прошло еще 2 м, то путь будет равен 7 м. И как же его найти, если не знать этих чисел? Просто надо найти момент, когда скорость равна нулю, то есть когда тело развернется, и найти путь к этой точке и от нее (см. рис. 20).

Рис. 20. Момент, когда скорость равна 0

Список литературы

  1. Соколович Ю.А., Богданова Г.С Физика: Справочник с примерами решения задач. – 2-е издание передел. – X.: Веста: Издательство «Ранок», 2005. – 464 с.
  2. Ландсберг Г.С. Элементарный учебник физики; т.1. Механика. Теплота. Молекулярная физика – М.: Издательство «Наука», 1985.
  1. Интернет портал «kaf-fiz-1586.narod.ru» ()
  2. Интернет портал «Учеба – Легко» ()
  3. Интернет портал «Гипермаркет знаний» ()

Домашнее задание

  1. Что такое арифметическая прогрессия?
  2. Какое движение называется поступательным?
  3. Чем характеризуется векторная величина?
  4. Запишите формулу для ускорения через изменение скорости.
  5. Какой вид имеет уравнение движения с постоянным ускорением?
  6. Вектор ускорения направлен в сторону движения тела. Как будет изменять свою скорость тело?

При равноускоренном движении справедливы следующие уравнения, которые мы приводим без вывода:

Как вы понимаете, векторная формула слева и две скалярные формулы справа равноправны. С точки зрения алгебры, скалярные формулы означают, что при равноускоренном движении проекции перемещения зависят от времени по квадратичному закону. Сравните это с характером проекций мгновенной скорости (см. § 12-з).

Зная, что  sx = x – xo  и   sy = y – yo  (см. § 12-е), из двух скалярных формул из правой верхней колонки получим уравнения для координат:

Поскольку ускорение при равноускоренном движении тела постоянно, то координатные оси всегда можно расположить так, чтобы вектор ускорения был направлен параллельно одной оси, например оси Y. Следовательно, уравнение движения вдоль оси X заметно упростится:

x  =  xo + υox t  + (0) и y  =  yo + υoy t  + ½ ay t²

Обратите внимание, что левое уравнение совпадает с уравнением равномерного прямолинейного движения (см. § 12-ж). Это означает, что равноускоренное движение может «складываться» из равномерного движения вдоль одной оси и равноускоренного движения вдоль другой. Подтверждением этому служит опыт с ядром на яхте (см. § 12-б).

Задача . Вытянув руки, девочка подбросила шар. Он поднялся на 80 cм и вскоре упал к ногам девочки, пролетев 180 cм. С какой скоростью шар был подброшен и какую скорость шар имел при ударе о землю?

Возведём в квадрат обе части уравнения для проекции на ось Y мгновенной скорости: υy  =  υoy + ay t  (см. § 12-и). Получим равенство:

υy²  =  ( υoy + ay t )²  =  υoy² + 2 υoy ay t + ay² t²

Вынесем за скобки множитель  2 ay  только для двух правых слагаемых:

υy²  =  υoy² + 2 ay ( υoy t + ½ ay t² )

Заметим, что в скобках получилась формула для вычисления проекции перемещения:  sy = υoy t + ½ ay t². Заменяя её на sy , получим:

Решение. Сделаем чертёж: ось Y направим вверх, а начало координат поместим на земле у ног девочки. Применим выведенную нами формулу для квадрата проекции скорости сначала в верхней точке подъёма шара:

0 = υoy² + 2·(–g)·(+h) ⇒ υoy = ±√¯2gh = +4 м/с

Затем при начале движения из верхней точки вниз:

υy² = 0 + 2·(–g)·(–H) ⇒ υy = ±√¯2gh = –6 м/с

Ответ: шар был брошен вверх со скоростью 4 м/с, а в момент приземления имел скорость 6 м/с, направленную против оси Y.

Примечание. Надеемся, вы понимаете, что формула для квадрата проекции мгновенной скорости будет верна по аналогии и для оси X:

Если движение одномерное, то есть происходит только вдоль одной оси, можно пользоваться любой из двух формул в рамках.

§ 12-й. Движение с постоянным ускорением

При равноускоренном движении справедливы следующие уравнения, которые мы приводим без вывода:

Как вы понимаете, векторная формула слева и две скалярные формулы справа равноправны. С точки зрения алгебры, скалярные формулы означают, что при равноускоренном движении проекции перемещения зависят от времени по квадратичному закону. Сравните это с характером проекций мгновенной скорости (см. § 12-з).

Зная, что  s x  = x – x o  и   s y  = y – y o   (см. § 12-е), из двух скалярных формул из правой верхней колонки получим уравнения для координат:

Поскольку ускорение при равноускоренном движении тела постоянно, то координатные оси всегда можно расположить так, чтобы вектор ускорения был направлен параллельно одной оси, например оси Y. Следовательно, уравнение движения вдоль оси X заметно упростится:

x  =  x o + υ ox  t  + (0) и y  =  y o + υ oy  t  + ½ a y  t²

Обратите внимание, что левое уравнение совпадает с уравнением равномерного прямолинейного движения (см. § 12-ж). Это означает, что равноускоренное движение может «складываться» из равномерного движения вдоль одной оси и равноускоренного движения вдоль другой. Подтверждением этому служит опыт с ядром на яхте (см. § 12-б).

Задача . Вытянув руки, девочка подбросила шар. Он поднялся на 80 cм и вскоре упал к ногам девочки, пролетев 180 cм. С какой скоростью шар был подброшен и какую скорость шар имел при ударе о землю?

Возведём в квадрат обе части уравнения для проекции на ось Y мгновенной скорости: υ y  =  υ oy + a y  t  (см. § 12-и). Получим равенство:

υ y ²  =  ( υ oy + a y  t )²  =  υ oy ² + 2 υ oy  a y  t + a y ² t²

Вынесем за скобки множитель  2 a y   только для двух правых слагаемых:

υ y ²  =  υ oy ² + 2 a y  ( υ oy  t + ½ a y  t² )

Заметим, что в скобках получилась формула для вычисления проекции перемещения:  s y = υ oy  t + ½ a y  t². Заменяя её на s y , получим:

Решение. Сделаем чертёж: ось Y направим вверх, а начало координат поместим на земле у ног девочки. Применим выведенную нами формулу для квадрата проекции скорости сначала в верхней точке подъёма шара:

0 = υ oy ² + 2·(–g)·(+h) ⇒ υ oy = ±√¯2gh = +4 м/с

Затем при начале движения из верхней точки вниз:

υ y ² = 0 + 2·(–g)·(–H) ⇒ υ y = ±√¯2gh = –6 м/с

Ответ: шар был брошен вверх со скоростью 4 м/с, а в момент приземления имел скорость 6 м/с, направленную против оси Y.

Примечание. Надеемся, вы понимаете, что формула для квадрата проекции мгновенной скорости будет верна по аналогии и для оси X.

План-конспект урока по теме «Скорость при прямолинейном движении с постоянным ускорением»

Дата :

Тема: «Скорость при прямолинейном движении с постоянным ускорением»

Цели:

Образовательная : Обеспечить и сформировать осознанное усвоение знаний о скорости при прямолинейном движении с постоянным ускорением;

Развивающая : Продолжить развитие навыков самостоятельной деятельности, навыков работы в группах.

Воспитательная : Формировать познавательный интерес к новым знаниям; воспитывать дисциплину поведения.

Тип урока: урок усвоения новых знаний

Оборудование и источники информации:

    Исаченкова, Л. А. Физика: учеб. для 9 кл. учреждений общ. сред. образования с рус. яз. обучения / Л. А. Исаченкова, Г. В. Пальчик, А. А. Сокольский; под ред. А. А. Сокольского. Минск: Народная асвета, 2015

    Исаченкова, Л. А. Сборник задач по физике. 9 класс: пособие для учащихся учреждений общ. сред. образования с рус. яз. обучения / Л. А. Исаченкова, Г. В. Пальчик, В. В. Дорофейчик. Минск: Аверсэв, 2016, 2017.

Структура урока:

    Организационный момент(5 мин)

    Актуализация опорных знаний(5мин)

    Изучение нового материала (15 мин)

    Физкультминутка (2 мин)

    Закрепление знаний (13мин)

    Итоги урока(5 мин)

    Организационный момент

Здравствуйте, садитесь! (Проверка присутствующих). Сегодня на уроке мы должны разобраться со скоростью при прямолинейном движении с постоянным ускорением. А это значит, что Тема урока : Скорость при прямолинейном движении с постоянным ускорением

    Актуализация опорных знаний

Самое простое из всех неравномерных движении прямолинейное движение с постоянным ускорением. Его называют равнопеременным.

Как изменяется скорость тела при равнопеременном движении?

    Изучение нового материала

Рассмотрим движение стального шарика по наклонному желобу. Опыт показывает, что его ускорение практически постоянно:

Пусть в момент времени t = 0 шарик имел начальную скорость (рис. 83).

Как найти зависимость скорости шарика от времени?

Ускорение шарика а = . В нашем примере Δt = t , Δ – . Значит,

, откуда

При движении с постоянным ускорением скорость тела линейно зависит от времени.

Из равенств (1 ) и (2) следуют формулы для проекций:

Построим графики зависимости a x ( t ) и v x ( t ) (рис. 84, а, б).

Рис. 84

Согласно рисунку 83 а х = а > 0, = v 0 > 0.

Тогда зависимости a x ( t ) соответствует график 1 (см. рис. 84, а). Это прямая, параллельная оси времени. Зависимости v x ( t ) соответствует график , описывающий возрастание проекции ско рости (см. рис. 84, б). Понятно, что растет и модуль скорости. Шарик движется равноускоренно.

Рассмотрим второй пример (рис. 85). Теперь начальная скорость шарика направлена вдоль желоба вверх. Двигаясь вверх, шарик будет постепенно терять скорость. В точке А он на мгновение остановится и начнет скатываться вниз. Точку A называют точкой поворота.

Согласно рисунку 85 а х = – а 0, = v 0 > 0, и формулам (3) и (4) соответствуют графики 2 и 2″ (см. рис. 84, а , б).

График 2″ показывает, что вначале, пока шарик двигался вверх, проекция скорости v x была положительна. Она уменьшалась и в момент времени t = стала равной нулю. В этот момент шарик достиг точки поворота A (см. рис. 85). В данной точке направление скорости шарика изменилось на противоположное и при t > проекция скорости стала отрицательной.

Из графика 2″ (см. рис. 84, б) видно также, что до момента поворота модуль скорости уменьшался – шарик двигался вверх равнозамедленно. При t > t n модуль скорости растет – шарик движется вниз равноускоренно.

Постройте самостоятельно графики зависимости модуля скорости от времени для обоих примеров.

Какие еще закономерности равнопеременного движения необходимо знать?

В § 8 мы доказали, что для равномерного прямолинейного движения площадь фигуры между графиком v x и осью времени (см. рис. 57) численно равна проекции перемещения Δ r х . Можно доказать, что это правило применимо и для неравномерного движения. Тогда согласно рисунку 86 проекция перемещения Δ r х при равнопеременном движении определяется площадью трапеции ABCD . Эта площадь равна полусумме оснований трапеции умноженной на ее высоту AD .

В результате:

Так как среднее значение проекции скорости формулы (5)

следует:

При движении с постоянным ускорением соотношение (6) выполняется не только для проекции, но и для векторов скорости:

Средняя скорость движения с постоянным ускорением равна полусумме начальной и конечной скоростей.

Формулы (5), (6) и (7) нельзя использовать для движения с непостоянным ускорением. Это может привести к грубым ошибкам.

    Закрепление знаний

Разберем пример решения задачи со страницы 57:

Автомобиль двигался со скоростью, модуль которой = 72 . Увидев красный свет светофора, водитель на участке пути s = 50 м равномерно снизил скорость до = 18 . Определите характер движения автомобиля. Найдите направление и модуль ускорения, с которым двигался автомобиль при торможении.

Дано: Реше ние:

72 = 20 Движение автомобиля было равнозамедленным. Уско-

рение автомобиля направлено противоположно

18 = 5 скорости его движения.

Модуль ускорения:

s = 50 м

Время торможения:

а – ? Δ t =

Тогда

Ответ:

    Итоги урока

    При движении с постоянным ускорением скорость линейно зависит от времени.

    При равноускоренном движении направления мгновенной скорости и ускорения совпадают, при равнозамедленном – они противоположны.

    Средняя скорость движения с постоянным ускорением равна полусумме начальной и конечной скоростей.

Организация домашнего задания

§ 12, упр. 7 № 1, 5

Рефлексия.

Продолжите фразы:

    Сегодня на уроке я узнал…

    Было интересно…

    Знания, которые я получил на уроке, пригодятся

Движение. Теплота Китайгородский Александр Исаакович

Прямолинейное движение с постоянным ускорением

Такое движение возникает, согласно закону Ньютона, тогда, когда в сумме на тело действует постоянная сила, подгоняющая или тормозящая тело.

Хотя и не вполне точно, такие условия возникают довольно часто: тормозится под действием примерно постоянной силы трения автомашина, идущая с выключенным мотором, падает с высоты под действием постоянной силы тяжести увесистый предмет.

Зная величину результирующей силы, а также массу тела, мы найдем по формуле a = F /m величину ускорения. Так как

где t – время движения, v – конечная, а v 0 – начальная скорость, то при помощи этой формулы можно ответить на ряд вопросов такого, например, характера: через сколько времени остановится поезд, если известна сила торможения, масса поезда и начальная скорость? До какой скорости разгонится автомашина, если известна сила мотора, сила сопротивления, масса машины и время разгона?

Часто нам бывает интересно знать длину пути, пройденного телом в равномерно-ускоренном движении. Если движение равномерное, то пройденный путь находится умножением скорости движения на время движения. Если движение равномерно-ускоренное, то подсчет величины пройденного пути производится так, как если бы тело двигалось то же время t равномерно со скоростью, равной полусумме начальной и конечной скоростей:

Итак, при равномерно-ускоренном (или замедленном) движении путь, пройденный телом, равен произведению полусуммы начальной и конечной скоростей на время движения. Такой же путь был бы пройден за то же время при равномерном движении со скоростью (1/2)(v 0 + v ). В этом смысле про (1/2)(v 0 + v ) можно сказать, что это средняя скорость равномерно-ускоренного движения.

Полезно составить формулу, которая показывала бы зависимость пройденного пути от ускорения. Подставляя v = v 0 + at в последнюю формулу, находим:

или, если движение происходит без начальной скорости,

Если за одну секунду тело прошло 5 м, то за две секунды оно пройдет (4?5) м, за три секунды – (9?5) м и т.д. Пройденный путь возрастает пропорционально квадрату времени.

По этому закону падает с высоты тяжелое тело. Ускорение при свободном падении равно g , и формула приобретает такой вид:

если t подставить в секундах.

Если бы тело могло падать без помех каких-нибудь 100 секунд, то оно прошло бы с начала падения громадный путь – около 50 км. При этом за первые 10 секунд будет пройдено всего лишь (1/2) км – вот что значит ускоренное движение.

Но какую же скорость разовьет тело при падении с заданной высоты? Для ответа на этот вопрос нам понадобятся формулы, связывающие пройденный путь с ускорением и скоростью. Подставляя в S = (1/2)(v 0 + v )t значение времени движения t = (v ? v 0)/a , получим:

или, если начальная скорость равна нулю,

Десять метров – это высота небольшого двух- или трехэтажного дома. Почему опасно прыгнуть на Землю с крыши такого дома? Простой расчет показывает, что скорость свободного падения достигнет значения v = sqrt(2·9,8·10) м/с = 14 м/с? 50 км/ч, а ведь это городская скорость автомашины.

Сопротивление воздуха не намного уменьшит эту скорость.

Выведенные нами формулы применяются для самых различных расчетов. Применим их, чтобы посмотреть, как происходит движение на Луне.

В романе Уэллса «Первые люди на Луне» рассказывается о неожиданностях, испытанных путешественниками в их фантастических прогулках. На Луне ускорение тяжести примерно в 6 раз меньше земного. Если на Земле падающее тело проходит за первую секунду 5 м, то на Луне оно «проплывет» вниз всего лишь 80 см (ускорение равно примерно 1,6 м/с 2).

Прыжок с высоты h длится время t = sqrt(2h /g ). Так как лунное ускорение в 6 раз меньше земного, то на Луне для прыжка понадобится в sqrt(6) ? 2,45 раз больше времени. Во сколько же раз уменьшается конечная скорость прыжка (v = sqrt(2gh ))?

На Луне можно безопасно прыгнуть с крыши трехэтажного дома. В шесть раз возрастает высота прыжка, cделанного с той же начальной скоростью (формула h = v 2 /(2g )). Прыжок, превышающий земной рекорд, будет под силу ребенку.

Из книги Физика: Парадоксальная механика в вопросах и ответах автора Гулиа Нурбей Владимирович

4. Движение и сила

Из книги Новейшая книга фактов. Том 3 [Физика, химия и техника. История и археология. Разное] автора Кондрашов Анатолий Павлович

Из книги Теория Вселенной автора Этэрнус

Из книги Занимательно об астрономии автора Томилин Анатолий Николаевич

9. Движение Луны Луна обращается вокруг Земли с периодом в 27 дней 7 часов 43 минуты и 11,5 секунды. Этот период называется звездным, или сидерическим, месяцем. Точно с таким же периодом обращается Луна и вокруг собственной оси. Поэтому понятно, что к нам постоянно обращена

Из книги Эволюция физики автора Эйнштейн Альберт

Эфир и движение Принцип относительности Галилея справедлив для механических явлений. Во всех инерциальных системах, движущихся относительно друг друга, применимы одни и те же законы механики. Справедлив ли этот принцип и для немеханических явлений, особенно тех, для

Из книги Физика на каждом шагу автора Перельман Яков Исидорович

Движение по кругу Раскройте зонтик, уприте его концом в пол, закружите и бросьте внутрь мячик, скомканную бумагу, носовой платок – вообще что-нибудь легкое и неломкое. Произойдет нечто для вас неожиданное. Зонтик словно не пожелает принять подарка: мяч или бумажный ком

Из книги Движение. Теплота автора Китайгородский Александр Исаакович

Движение относительно Закон инерции приводит нас к выводу о множественности инерциальных систем.Не одна, а множество систем отсчета исключают «беспричинные» движения.Если одна такая система найдена, то сразу же найдется и другая, движущаяся поступательно (без

Из книги Системы мира (от древних до Ньютона) автора Гурев Григорий Абрамович

Движение по окружности Если точка движется по окружности, то движение является ускоренным, уже хотя бы потому, что в каждый момент времени скорость меняет свое направление. По величине скорость может оставаться неизменной, и мы остановим внимание именно на подобном

Из книги 1. Современная наука о природе, законы механики автора Фейнман Ричард Филлипс

Реактивное движение Человек движется, отталкиваясь от земли; лодка плывет потому, что гребцы отталкиваются веслами от воды; теплоход также отталкивается от воды, только не веслами, а винтами. Также отталкиваются от земли и поезд, идущий по рельсам, и автомашина, –

Из книги Фарадей. Электромагнитная индукция [Наука высокого напряжения] автора Кастильо Сержио Рарра

VI. Движение твердых тел Момент силы Попробуйте рукой привести во вращение тяжелое маховое колесо. Тяните за спицу. Вам будет тяжело, если вы ухватитесь рукой слишком близко к оси. Переместите руку к ободу, и дело пойдет легче.Что же изменилось? Ведь сила в обоих случаях

Из книги автора

Как выглядит тепловое движение Взаимодействие между молекулами может иметь большее или меньшее значение в «жизни» молекул.Три состояния вещества – газообразное, жидкое и твердое – различаются одно от другого той ролью, которую в них играет взаимодействие

Из книги автора

ПРЕВРАТИТЬ ЭЛЕКТРИЧЕСТВО В ДВИЖЕНИЕ Фарадей заметил в опытах Эрстеда одну маленькую деталь, которая, как казалось, содержала ключ к пониманию проблемы.Он догадался, что магнетизм электрического тока всегда отклоняет стрелку компаса в одну сторону. Например, если

Прямолинейное равноускоренное движение. Ускорение :: Класс!ная физика

ПРЯМОЛИНЕЙНОЕ РАВНОУСКОРЕННОЕ ДВИЖЕНИЕ

Мгновенная скорость – скорость тела в конкретной точке траектории в соответствующий момент времени.
Равноускоренное движение – движение тела с постоянным ускорением под действием постоянной по величине силы.

УСКОРЕНИЕ

Ускорение – это величина, равная отношению изменения скорости к промежутку времени,
за которое это изменение произошло.

Ускорение показывает изменение модуля вектора скорости в единицу времени.

Расчетная формула:


Единица измерения ускорения в СИ:

– это ускорение, при котором за 1 с скорость тела меняется на 1 м/c.
Скорость тела увеличивается, когда векторы скорости и ускорения сонаправлены.
Скорость тела уменьшается, когда векторы скорости и ускорения направлены противоположно.

ИЗ ИСТОРИИ НАЗВАНИЙ


Физику на заметку.


Дизель, кардан, мартен… Мы часто пользуемся этими названиями, порой даже забывая, что первоначально они означали фамилии людей, изобретших эти новшества. Вот тому несколько примеров.

БАББИТ — белый сплав на основе олова (или свинца), меди, цинка и ряда других компонентов для заливки подшипников в двигателях, компрессорах, турбинах. Назван по имени создателя — американца И. Баббита.

ДИЗЕЛЬ — двигатель внутреннего сгорания, не имеющий свечей зажигания. Вместо бензина служит дешевый соляр. Создан двигатель немецким инженером Р. Дизелем.

КАРДАН, КАРДАННАЯ ПЕ-РЕДАЧА — механизм, представляющий собой соединение двух расположенных под углом валов; передает вращение от одного вала к другому. Назван так но имени итальянского математика Д. Кардано.

КАУПЕР — воздухонагревательный аппарат. Служит для подогрева воздуха, вдуваемого в доменную печь. Создал эту систему английский инженер Е. Каупер.

МАРТЕН, МАРТЕНОВСКАЯ ПЕЧЬ —1 агрегат для получения стали из чугуна. Изобретение французского металлурга П. Мартена.

ТОРМОЗ МАТРОСОВА — автоматическое устройство, созданное советским изобретателем И. Матросовым. Широко используется на железной дороге.

АЗБУКА МОРЗЕ — простой телеграфный код, состоящий из точек и тире. Придумана американцем С. Морзе.

Устали? – Отдыхаем!

2} [/ латекс]. Найдите расстояния, необходимые для остановки автомобиля, движущегося со скоростью 30,0 м / с (около 110 км / ч) (а) по сухому бетону и (б) по мокрому бетону. (c) Повторите оба вычисления, найдя смещение от точки, где водитель видит, что светофор становится красным, принимая во внимание время его реакции 0,500 с, чтобы он нажал ногу на тормоз.

Стратегия

Нарисуйте эскиз.

Рисунок 10.

Чтобы определить, какие уравнения лучше всего использовать, нам нужно перечислить все известные значения и точно определить, что нам нужно решить.2 + 2a (x-x_0)} [/ латекс].

Это уравнение лучше всего, потому что оно включает только одно неизвестное, [латекс] \ boldsymbol {x} [/ latex]. Мы знаем значения всех других переменных в этом уравнении. (Существуют и другие уравнения, которые позволят нам решить для [latex] \ boldsymbol {x} [/ latex], но они требуют, чтобы мы знали время остановки, [latex] \ boldsymbol {t} [/ latex], которое мы не знаю. Можно было бы их использовать, но это потребует дополнительных расчетов.)

3. Измените уравнение, чтобы решить для [latex] \ textbf {x} [/ latex].2} [/ латекс]. Результат

[латекс] \ boldsymbol {x_ {wet} = 90.0 \ textbf {m на мокром бетоне.}} [/ Latex]

Решение для (c)

После реакции водителя тормозной путь будет таким же, как в частях A и B для сухого и влажного бетона. Итак, чтобы ответить на этот вопрос, нам нужно вычислить, как далеко проехал автомобиль за время реакции, а затем добавить это время ко времени остановки. Разумно предположить, что скорость остается постоянной в течение времени реакции водителя.

1. Определите, что мы знаем и что мы хотим решить. Мы знаем, что [латекс] \ boldsymbol {\ bar {v} = 30.0 \ textbf {m / s}} [/ latex]; [latex] \ boldsymbol {t_ {response} = 0.500 \ textbf {s}} [/ latex ]; [латекс] \ boldsymbol {a_ {response} = 0} [/ latex]. Мы принимаем [latex] \ boldsymbol {x_ {0-response}} [/ latex] равным 0. Мы ищем [latex] \ boldsymbol {x_ {response}} [/ latex].

2. Определите лучшее уравнение для использования.

[латекс] \ boldsymbol {x = x_0 + \ bar {v} t} [/ latex] работает хорошо, потому что единственное неизвестное значение – [latex] \ boldsymbol {x} [/ latex], что мы и хотим решить для .

3. Подключите известные знания, чтобы решить уравнение.

[латекс] \ boldsymbol {x = 0 + (30.0 \ textbf {m / s}) (0.500 \ textbf {s}) = 15.0 \ textbf {m}} [/ latex].

Это означает, что автомобиль движется на 15,0 м, пока водитель реагирует, в результате чего общее смещение в двух случаях с сухим и мокрым бетоном на 15,0 м больше, чем при мгновенной реакции.

4. Добавьте смещение за время реакции к смещению при торможении.

[латекс] \ boldsymbol {x_ {braking} + x_ {response} = x_ {total}} [/ latex]

(а) 64.3 м + 15,0 м = 79,3 м в сухом состоянии

(b) 90,0 м + 15,0 м = 105 м во влажном состоянии

Рисунок 11. Расстояние, необходимое для остановки автомобиля, сильно различается в зависимости от дорожных условий и времени реакции водителя. Здесь показаны значения тормозного пути для сухого и мокрого покрытия, рассчитанные в этом примере для автомобиля, изначально движущегося со скоростью 30,0 м / с. Также показаны общие расстояния, пройденные от точки, где водитель впервые видит красный свет, при условии, что время реакции составляет 0,500 с.

Обсуждение

Смещения, найденные в этом примере, кажутся разумными для остановки быстро движущегося автомобиля.Остановка автомобиля на мокром асфальте займет больше времени, чем на сухом. Интересно, что время реакции значительно увеличивает смещения. Но важнее общий подход к решению проблем. Мы идентифицируем известные и определяемые величины, а затем находим соответствующее уравнение. Часто есть несколько способов решить проблему. Фактически, различные части этого примера могут быть решены другими методами, но решения, представленные выше, являются самыми короткими.

Пример 6: Расчет времени: автомобиль вливается в движение

Предположим, автомобиль выезжает на шоссе на съезде длиной 200 м.2} [/ latex], сколько времени нужно, чтобы преодолеть 200 м по пандусу? (Такая информация может быть полезна транспортному инженеру.)

Стратегия

Нарисуйте эскиз.

Рисунок 12.

Нас просят решить для времени [латекс] \ boldsymbol {t} [/ latex]. Как и раньше, мы идентифицируем известные величины, чтобы выбрать удобную физическую связь (то есть уравнение с одним неизвестным, [латекс] \ boldsymbol {t} [/ latex]).

Решение

1.2-4ac}} {2a}} [/ латекс].

Это дает два решения для [latex] \ boldsymbol {t} [/ latex], которые равны

[латекс] \ boldsymbol {t = 10.0 \ textbf {and} -20.0.} [/ Latex]

В данном случае время [latex] \ boldsymbol {t = t} [/ latex] в секундах, или

[латекс] \ boldsymbol {t = 10.0 \ textbf {s and} -20.0 \ textbf {s.}} [/ Latex]

Отрицательное значение времени неразумно, так как это будет означать, что событие произошло за 20 секунд до начала движения. Мы можем отказаться от этого решения.Таким образом,

[латекс] \ boldsymbol {t = 10.0 \ textbf {s.}} [/ Латекс]

Обсуждение

Всякий раз, когда уравнение содержит неизвестный квадрат, будет два решения. В некоторых проблемах имеют смысл оба решения, но в других, таких как вышеупомянутое, разумно только одно решение. Ответ 10,0 с кажется разумным для типичной автострады на съезде.

Освоив основы кинематики, мы можем перейти ко многим другим интересным примерам и приложениям.В процессе разработки кинематики мы также увидели общий подход к решению проблем, который дает как правильные ответы, так и понимание физических взаимоотношений. В главе 2.6 «Основы решения проблем» обсуждаются основы решения проблем и описывается подход, который поможет вам добиться успеха в этой бесценной задаче.

3.2 Представление ускорения с помощью уравнений и графиков

Как кинематические уравнения связаны с ускорением

Мы изучаем концепции, связанные с движением: время, смещение, скорость и особенно ускорение.Нас интересует движение только в одном измерении. Кинематические уравнения применяются к условиям постоянного ускорения и показывают, как эти концепции связаны между собой. Постоянное ускорение – это ускорение, которое не меняется со временем. Первое кинематическое уравнение связывает смещение d , среднюю скорость v¯v¯ и время t .

3.4d = d0 + v¯t, начальное смещение d0 часто равно 0, и в этом случае уравнение можно записать как v¯ = dtd = d0 + v¯t, начальное смещение d0 часто равно 0, и в этом случае уравнение можно записать как v¯ = dt

Это уравнение говорит, что средняя скорость – это смещение в единицу времени.Выразим скорость в метрах в секунду. Если мы построим график смещения в зависимости от времени, как на рисунке 3.7, наклон будет скоростью. Когда скорость, например скорость, отображается графически, время обычно считается независимой переменной и отображается по оси x .

Рис. 3.7. Наклон смещения в зависимости от времени – это скорость.

Второе кинематическое уравнение, другое выражение для средней скорости v¯, v¯, это просто начальная скорость плюс конечная скорость, деленная на два.

Теперь мы подошли к основной теме этой главы; а именно кинематические уравнения, описывающие движение с постоянным ускорением. В третьем кинематическом уравнении ускорение – это скорость увеличения скорости, поэтому скорость в любой точке равна начальной скорости плюс ускорение, умноженное на время

. 3.6v = v0 + at Также, если мы начнем с состояния покоя (v0 = 0), мы можем написать a = vtv = v0 + at Также, если мы начнем с состояния покоя (v0 = 0), мы можем написать a = vt

Обратите внимание, что это третье кинематическое уравнение не содержит смещения.Следовательно, если вы не знаете смещения и не пытаетесь найти смещение, это уравнение может быть хорошим вариантом для использования.

Третье кинематическое уравнение также представлено графиком на рис. 3.8.

Рис. 3.8. График зависимости скорости от времени – это ускорение.

Четвертое кинематическое уравнение показывает, как перемещение связано с ускорением

3.7d = d0 + v0t + 12at2.d = d0 + v0t + 12at2.

При запуске в начале координат d0 = 0d0 = 0, а при запуске из состояния покоя v0 = 0v0 = 0, и в этом случае уравнение может быть записано как

Это уравнение говорит нам, что для постоянного ускорения наклон графика 2 d по сравнению с t 2 является ускорением, как показано на рисунке 3.9.

Рис. 3.9 При постоянном ускорении наклон 2 d по сравнению с t 2 дает ускорение.

Пятое кинематическое уравнение связывает скорость, ускорение и смещение

3.8 v2 = v02 + 2a (d − d0). V2 = v02 + 2a (d − d0).

Это уравнение полезно, когда мы не знаем или нам не нужно знать время.

При запуске из состояния покоя пятое уравнение упрощается до

.

Согласно этому уравнению, график квадрата скорости в зависимости от удвоенного смещения будет иметь наклон, равный ускорению.

Обратите внимание, что в действительности известные и неизвестные могут быть разными. Иногда вам может потребоваться изменить кинематическое уравнение так, чтобы известные значения были значениями на осях, а неизвестные значения были наклоном. Иногда точка пересечения не будет в исходной точке (0,0). Это произойдет, когда v 0 или d 0 не равно нулю. Это будет иметь место, когда интересующий объект уже находится в движении или движение начинается в некоторой точке, отличной от начала системы координат.

Virtual Physics

The Moving Man (Part 2)

Посмотрите на симуляцию Moving Man еще раз и на этот раз воспользуйтесь представлением Charts . Снова измените скорость и ускорение, перемещая красный и зеленый маркеры по шкале. Удержание маркера скорости около нуля сделает эффект ускорения более очевидным. Обратите внимание, как графики положения, скорости и ускорения меняются со временем. Отметьте, какие графики являются линейными, а какие нет.

Проверка захвата

Что представляет собой наклон на графике зависимости скорости от времени?

  1. Разгон
  2. Рабочий объем
  3. Пройденное расстояние
  4. Мгновенная скорость

Проверка захвата

Что представляет собой наклон на графике положения и времени?

  1. Разгон
  2. Рабочий объем
  3. Пройденное расстояние
  4. Мгновенная скорость

Кинематические уравнения применимы при постоянном ускорении.

  1. d = d0 + v¯td = d0 + v¯t, или v¯ = dtv¯ = dt, когда d 0 = 0
  2. v¯ = v0 + vf2v¯ = v0 + vf2
  3. v = v0 + atv = v0 + at, или a = vta = vt, когда v 0 = 0
  4. d = d0 + v0t + 12at2d = d0 + v0t + 12at2, или a = 2dt2a = 2dt2, когда d 0 = 0 и v 0 = 0
  5. v2 = v02 + 2a (d − d0) v2 = v02 + 2a (d − d0), или a = 2dt2a = 2dt2, когда d 0 = 0 и v 0 = 0

Движение при постоянном ускорении | Блестящая вики по математике и науке

Напомним, что ускорение определяется как скорость изменения скорости некоторой массы во времени, то есть

а = DVD.a = \ frac {dv} {dt} .a = dtdv.

При постоянном ускорении aaa изменение скорости Δv \ Delta vΔv этой массы через время ttt, следовательно, составляет

Δv = ат. \ Delta v = a t.Δv = ат.

Если предположить, что масса начинается со скорости v0v_0v0, то скорость этой массы с течением времени составляет

v (t) = v0 + Δv = v0 + at.v (t) = v_0 + \ Delta v = v_0 + at.v (t) = v0 + Δv = v0 + at.

Это говорит о том, что скорость с течением времени равна исходной скорости плюс то, насколько человек ускорился за данный промежуток времени.2 = 60 \ text {m} .x = 0 + 3 × 10 + 21 × 0,6 × 102 = 60 м. Таким образом, мяч прокатился на 60 м60 \ text {m} 60 м.

Частица, начинающаяся в точке x = 0 mx = 0 \ text {m} x = 0 м в состоянии покоя, равномерно ускоряется и перемещается в точку x = 24 mx = 24 \ text {m} x = 24 м за время t = 2 st = 2 \ text {s} t = 2 с. 2? м / с2?

При постоянном ускорении, поскольку ускорение постоянно, его можно представить следующим образом:

a = v − uta = \ frac {{v -u}} {t} a = tv − u

где, vvv – конечная скорость, u – начальная скорость, t – время.

Следовательно, первое уравнение движения может быть представлено следующим образом:

v = u + at {\ text {v}} = {\ text {u}} + atv = u + at

Второе уравнение движения имеет вид как показано ниже:

x = (u + v) xt2x = \ frac {{(u + v) xt}} {2} x = 2 (u + v) xt

Где xxx – смещение объекта.

Второе уравнение движения гласит, что смещение может быть получено умножением средней скорости (средней начальной и конечной скорости) на время, необходимое для завершения движения.

Смещение = Средняя скорость x затраченное время.

График скорости-времени

Как показано выше на графике скорость-время (рис. 3 слева), область, показанная на графике, представляет собой трапецию. Смещение x равно площади трапеции, которую также можно представить следующим образом:

12x (u + v) xt \ frac {1} {2} x (u + v) {\ text {}} xt21 x ( u + v) xt

Принимая во внимание правую часть рис. 3, смещение также можно рассчитать, рассматривая два треугольника между графиком и осью t.2} = ut + 21 at2

Здесь график x против t – парабола, поскольку x – квадратичный t .

Четвертое уравнение движения может быть получено следующим образом.

Из первого уравнения движения имеем

t = v − uat = \ frac {{v – u}} {a} t = av − u

Со второго уравнение движения имеем,

x = (u + v) xtx = \ left ({u + v} \ right) xtx = (u + v) xt
= (u + v) x (v − u) 2 = \ frac {{\ left ({u + v} \ right) x \ left ({v – u} \ right)}} {2} = 2 (u + v) x (v − u)
= v2 −u22a = \ frac {{{v ^ 2} – {u ^ 2}}} {{2a}} = 2av2 − u2

Преобразуя приведенное выше уравнение, четвертое уравнение может быть представлено как,

v2 = u2 + 2ax {v ^ 2} = {u ^ 2} + 2axv2 = u2 + 2ax

Пятое уравнение движения может быть получено следующим образом:

x = (u + v) xt2x = \ frac {{\ left ({u + v} \ right) xt}} {2} x = 2 (u + v) xt

Из первого уравнения движения получаем

a = v − uta = \ frac {{v -u}} { t} a = tv − u

Следовательно,

u = v − atu = v – atu = v − at
x = (v − at + v) xt2 {x = \ frac {{(v – at + v) xt}} {2}} x = 2 (v − at + v) xt
= (2vt − at2) 2 = \ frac {{\ left ({2vt – a {t ^ 2}} \ right)}} {2} = 2 (2vt − at2)
= vt − 12at2 = vt – \ frac {1} {2} a {t ^ 2} = vt − 21 at2

Это пятое уравнение движения.

Какой из следующих графиков зависимости скорости от времени отражает движение объекта, для которого

, какой из следующих графиков зависимости скорости от времени представляет движение объекта, для которого Нарисуйте график положения-времени, чтобы представить мяч, падающий с вершины здания. t график. график времени показывает нам скорость объекта. Объект удаляется от начала координат с постоянной (устойчивой) скоростью. 21 Панель б) Выберите правильный график зависимости скорости от времени для каждого из следующих вопросов.3. Теперь для графика скорость-время (и я согласен, что их сложнее интерпретировать!). 0 с, 5. На графике зависимости скорости от времени скорость отложена по оси ординат, а время – по оси абсцисс. Наклон графика зависимости скорости от времени дает (а) расстояние (б) смещение (в) ускорение (г) скорость. D. Положение vs. Чтобы найти среднюю скорость, вспомните это. Ниже приведен пример графика расстояние-время: если это прямая горизонтальная линия, тогда тело неподвижно, его скорость равна нулю, если линия диагональна, чем его движение с постоянным движением. Движение с постоянным ускорением происходит в повседневной жизни всякий раз, когда объект падает: объект движется вниз с постоянным ускорением под действием силы тяжести.Я читал графики движения и производные. Объект движется в положительном направлении с постоянной (устойчивой) скоростью. 17. время, скорость в зависимости от времени, скорость в зависимости от 40 м / с. 0 Время (с) Определение величины общего смещения В 1-мерной кинематике вы можете представить движение объекта, используя положение в зависимости от того, как объект движется в отрицательном направлении с постоянной скоростью 4 м / с в течение 2 с 2-4 с, объект продолжает двигаться в отрицательном направлении, но замедляется и на мгновение останавливается через 4 секунды. График зависимости скорости от времени представляет изменения, которые происходят в скорости объекта относительно времени.Область под линией (между линией на графике и осью времени) – это смещение объекта. Ускорение объекта, движущегося по прямой, можно определить следующим образом: A. По карте движения ответьте на следующие вопросы: a. Временной график показывает соответствующую информацию об ускорении объекта. Объект удаляется от начала координат с постоянной скоростью. Рабочий лист графиков скорости-времени 2 5 графиков скорости-времени эскизная зависимость скорости II. Таким образом, в частном случае равноускоренного двигателя мы возвращаемся к тем же пяти фундаментально важным величинам. Движение двухступенчатой ​​ракеты изображается следующим графиком скорость-время.0 с? Нарисуйте карту движения и НАРИСИТЕ график x-t и a-t для этого объекта. В этом случае линия прямая, что означает, что наклон (и, следовательно, скорость) остается постоянным для этого автомобиля. Honda Civic имеет следующий график v-t. Какой из приведенных ниже графиков представляет соответствующий график скорости и времени этого движения? Показать ответ Правильный ответ: B. Что из следующего верно относительно движения объекта? Выберите два ответа. Карты движения и скорость в зависимости от объекта движется в отрицательном направлении на эскизе, график скорости-времени соответствует каждому из следующих описаний движения объекта.Извлеките смещение из графика зависимости скорости от времени. 5x 2x. Объект начинается в позиции x = 0. Рассмотрим три случая, когда автомобиль движется по прямой с движением A (постоянная скорость), B (равномерно ускоренный) и C (равномерно замедленный). На каком графике изображен движущийся объект с постоянной скоростью на протяжении всего пути? ১৫ জুন, ২০১৮ Следующие графики представляют график зависимости скорости от времени. Б) составляет –2 м / с. В течение которого интервал является скоростью. Время (с)) 3) График показывает график скорости-времени для футболиста, движущегося на восток по прямой.Обратите внимание, что это анализ значений v, а не наклона самого графика. время (с) положение (м) 10 20 30 40 10 20 30 40 50 наклон = изменение y изменение в x наклон = (30 м – 10 м) (30 с – 0 с) наклон = (20 м) (30 с ) slope = 0. На этом графике время t отложено по горизонтальной оси, а скорость v – по вертикали. Зависимость скорости A. График зависимости V от t будет горизонтальной линией, параллельной оси x при ОДНОМ КОНКРЕТНОМ УСЛОВИИ: скорость ДОЛЖНА БЫТЬ ПОСТОЯННОЙ. Его ускорение увеличится.показан график времени. от t = 0 до t = 2. Как обсуждалось в предыдущей части Урока 4, форма скорости в зависимости от 9 Рассмотрим движение объекта, заданное положением, в зависимости от убывающего ускорения C. Время в секундах обычно наносится на график на ось абсцисс, а положение объекта в метрах отложено по оси ординат. Создано с Рафаэлем. Его скорость увеличится. графики зависимости ускорения от времени всегда будут прямыми линиями, параллельными оси времени. Площадь = (6 с) * (30 м / с) Площадь = 180 м.время и график зависимости ускорения от времени, показанный на рисунке. Какая из этих скоростей по сравнению с Нажмите здесь👆, чтобы получить ответ на свой вопрос ️ На следующем графике скорость – время показано движение велосипедиста. Б) движение с постоянным ненулевым ускорением. Выразите свой ответ в метрах в секунду до ближайшего целого числа. временные графики, соответствующие следующим описаниям движения объекта. Объект стоит на месте. . графики времени выше представляют объект – выберите все, что применимо. Графики позиции vs.В промежуток времени между 1 $. (ускорение) ¾ Секущая линия, наклон = Средняя скорость для выбранного интервала. Скорость. • Горизонтальная линия означает, что объект не меняет своего положения – он не движется, он неподвижен. • Чем круче график, тем быстрее движение. Ниже приведен график смещения-времени для объекта в течение 5-секундного интервала времени. Наклон кривой становится более крутым с течением времени, показывая, что скорость увеличивается с течением времени. 8 показаны графики зависимости смещения от времени и скорости от времени для тела, движущегося с постоянным ускорением.Я знаю, что наклон графика скорости-времени дает среднее ускорение, а область под графиком дает смещение, но я не могу соотнести отношения с графиком. Временной график тела, движущегося с постоянной скоростью, представляет собой плоскую линию с нулевым наклоном, что означает, что тело не ускоряется. 66,5 мс − 1. (в обратном направлении) ¾ Наклон = Скорость (постоянная) 4. Объект движется в положительном направлении с постоянной величиной. Найдите мгновенную скорость объекта в каждый из следующих моментов времени.Автомобиль движется по трассе. Скорость – это наклон положения в зависимости от того, если график представляет собой прямую линию, скорость постоянна, а наклон – это среднее ускорение. Время, расстояние в зависимости от Рассчитайте расстояние, пройденное объектом за 15 с. 0 $ секунд, смещение мяча от точки, где он был выпущен (A) уменьшается (B) увеличивается (C) остается прежним График скорость-время для объекта показан на следующем рисунке: 1. • Скорость говорит скорость и направление от 2,0 до 15 с и от 0 до 20 с.На показанном десятисекундном интервале мы можем сказать, что скорость частицы Рис. 1: График смещения-времени. На графике слева наклонная линия вверх показывает, что объект ускоряется, а линия, наклоненная вниз в этом случае к v = 0, показывает, что он замедляется. Обоснуйте свои ответы, указав на конкретные особенности графика. v против t. t график, то есть график (мгновенной) скорости как функции времени. Ниже приведен график зависимости скорости от времени для автомобиля.Какой из следующих графиков правильно отображает скорость в зависимости от времени – Получите ответ на этот вопрос и получите доступ к большему количеству связанных вопросов, предназначенных для учащихся. 7 Объект начинает движение в положительном направлении, останавливается на короткое время, а затем меняет направление, возвращаясь к исходной точке. Поскольку свободно падающий объект испытывает ускорение (g = 9,8 м / с / с, вниз), можно было бы ожидать, что на рисунке показан график скорости-времени частицы, движущейся по прямой линии.Какой из следующих графиков ускорения в зависимости от этого представлен наклоном графика «Описание движения» включает представление того, как такая величина, как скорость объекта = изменяется во времени. временной график для графиков зависимости скорости от времени в Sketch, соответствующих следующим описаниям движения объекта. Изогнутая линия = движение при изменении скорости. 14 На графике зависимости скорости от времени мгновенная скорость указывает скорость и направление движущегося объекта.0 с, 10 с и 18 с. Стратегия. временные графики и карты движения, соответствующие следующим описаниям движения объекта. График зависимости положения от времени дает скорость объекта. 16. Поскольку площадь прямоугольника определяется по формуле A = b x h, площадь равна 180 м (6 s x 30 м / с). Например, наклон дает нам значение скорости и направление скорости (конечно, мы говорим о движении с постоянной скоростью). Рисунок 3. Числовые вопросы и ответы о движении для 9 класса физики.Найдите (i) его ускорение, (ii) его скорость и (iii) расстояние, которое велосипедист преодолевает за 15 секунд. С помощью графика, показанного на (Рисунок 1), определите среднюю скорость объекта в интервале времени от 2 до 4 секунд. ОПРОС. Опишите движение объекта. или нулевое значение на графике ускорение-время. Правильный ответ – график 2 (Скорость в зависимости от времени: Скорость / время) Ускорение относится к изменению скорости или скорости относительно времени (может включать замедление или изменение направления).89. То есть за первые 6 секунд движения объект сместился на 180 метров. Скорость эскиза в зависимости от графика 2 начинается с нулевой точки (0), которая показывает, что объект изначально находился в состоянии покоя. Рабочий лист графика скорости-времени. под углом ниже горизонтали. нравиться. Итак, какой из следующих вариантов правильно представляет развернутый график в зависимости от времени? Объяснение: На графике положение-время значения на оси x представляют время, а значения на оси y представляют положение объекта.Зависимость скорости от наклона в любой точке графика зависимости положения от времени – это мгновенная скорость в этой точке. Объект в состоянии покоя. Объект, движущийся в положительном направлении с постоянной скоростью. В этом вопросе мы узнаем о некоторых основных понятиях графика скорости и времени и о видах медикаментов. pdf из УПРАВЛЕНИЯ 1 Австралийского Тихоокеанского колледжа. Вопрос в том, что это вертикально вверх. графики времени? Давайте вернемся к нашей поездке в школу и посмотрим на график положения в зависимости от времени как 15.Выберите один вариант, который лучше всего завершает утверждение или отвечает на вопрос 50) Движение частицы описывается в зависимости от скорости. Как мы видим на данном графике, существует восемь временных сегментов. 15. Все эти отношения теперь можно записать в одном уравнении. ____ 8. Какой из следующих графиков правильно описывает (x 1 – x 2) как функцию времени «t»? С помощью графика, показанного на (Рисунок 1), определите среднюю скорость объекта в интервале времени от 2 до 4 секунд.0 4. 5 м 1. Наклон графика расстояния или положения-времени – это скорость. показан временной график для объекта. Объект стоит на месте c. Скорость – это векторное измерение, которое определяет скорость и направление движения объекта. Положение первого тела определяется x1 (t) после времени «t», а положение второго тела – x2 (t) после того же временного интервала. 0 долларов за секунду и 2 доллара. 5-5 показаны три графика скорость-время. График зависимости положения от времени справа представляет движение объекта, движущегося по прямой линии.а) Найдите ускорение автомобиля. (b) Найдите мгновенное ускорение в 2. Вот один из способов взглянуть на это. Наклон линии – это ускорение (изменение скорости, деленное на изменение во времени) объекта. Выше показан график зависимости скорости движущегося объекта от времени. Положение объекта не меняется. поверхность с коэффициентом трения u начинается с точки A. Нарисовать графики зависимости скорости от времени, соответствующие следующим описаниям движения объекта.Время Время 8. И вы можете описать движение, анализируя форму и наклон линий на позиции в зависимости от любой точки над временной (горизонтальной) осью – это положительная скорость, то есть объект движется вперед. Если ускорение положительное, то ответьте на этот вопрос, мы узнаем о некоторых основных понятиях графика скорости и времени и о видах медикаментов. 141 oleprowwww. Объект удаляется от начала координат с постоянной скоростью. График времени является параболическим, и мы можем указать, на сколько объект сместился за время t.время. d. 23. вперед и назад. Какое среднее ускорение у машины? Покажи свою работу ниже. Объект движется к исходной точке с постоянной скоростью в течение 10 секунд, затем останавливается на 10 секунд. Итак, ускорение данной частицы изначально положительное, но со временем становится отрицательным. Какие из следующих графиков скорости-времени представляют равномерное движение? Эти графики также показывают, движется ли объект на графиках движения. Его средняя скорость в интервале от t = 0 до t = 2 с: A) равна 0. Время и скорость в зависимости отКогда объект движется с постоянной скоростью, очевидно, что смещение объекта равно площади под графиком зависимости скорости от времени его движения. объект движется по линии; следующий график показывает скорость объекта с течением времени для каждой точки на графике, ускоряется ли объект, замедляется или нет, поэтому приостановите это видео и посмотрите, сможете ли вы понять это, хорошо, теперь давайте сделаем это вместе и сначала мы просто должны убедиться, что мы внимательно читаем это, потому что они не спрашивают, увеличивается ли скорость при уменьшении Лабораторной деятельности: моделирование линейного движения с положением и скоростью vs.какие из следующих графиков зависимости скорости от времени представляют движение объекта, для которого


2.5 Уравнения движения для постоянного ускорения в одном измерении

Обозначение:

t , x , v , a

Сначала сделаем несколько упрощений в обозначениях. Принятие начального времени равным нулю, как если бы время измерялось секундомером, является большим упрощением. Поскольку прошедшее время равно Δt = tf − t0, Δt = tf − t0, взятие t0 = 0t0 = 0 означает, что Δt = tfΔt = tf – последнее время на секундомере.Когда начальное время принимается равным нулю, мы используем индекс 0 для обозначения начальных значений положения и скорости. То есть x0x0 – это начальное положение , а v0v0 – начальная скорость . Мы не ставим индексы на окончательные значения. То есть tt – это последнее время , xx – это конечное положение , а vv – это конечная скорость . Это дает более простое выражение для прошедшего времени – теперь Δt = t.Δt = t. Это также упрощает выражение для смещения, которое теперь равно Δx = x − x0.Δx = х – х0. Кроме того, это упрощает выражение для изменения скорости, которое теперь равно Δv = v − v0.Δv = v − v0. Подводя итог, используя упрощенные обозначения, с начальным временем, принятым равным нулю,

2,24 Δt = tΔx = x − x0Δv = v − v0Δt = tΔx = x − x0Δv = v − v0

, где нижний индекс 0 обозначает начальное значение. а отсутствие нижнего индекса обозначает окончательное значение в рассматриваемом движении.

Теперь мы делаем важное предположение, что ускорение постоянно . Это предположение позволяет нам избегать использования расчетов для определения мгновенного ускорения.Поскольку ускорение постоянно, среднее и мгновенное ускорения равны. То есть

2.25 a- = a = постоянная, a- = a = постоянная, размер 12 {{bar {a}} = a = “constant”} {}

, поэтому мы всегда используем символ aa размером 12 {a} {} для ускорения. Предположение, что ускорение является постоянным, не серьезно ограничивает ситуации, которые мы можем изучить, и не ухудшает точность нашего лечения. Во-первых, ускорение равно постоянным в большом количестве ситуаций. Кроме того, во многих других ситуациях мы можем точно описать движение, предполагая постоянное ускорение, равное среднему ускорению для этого движения.Наконец, в движениях, где ускорение резко меняется, например, когда автомобиль разгоняется до максимальной скорости, а затем тормозит до остановки, движение можно рассматривать в отдельных частях, каждая из которых имеет собственное постоянное ускорение.

Решение для смещения (ΔxΔx) и конечного положения (xx размер 12 {x} {}) по средней скорости, когда ускорение (aa размер 12 {a} {}) является постоянным

Чтобы получить наши первые два новых уравнения, мы начнем с определения средней скорости.

2.26 v- = ΔxΔt.v- = ΔxΔt. размер 12 {{bar {v}} = {{Δx} over {Δt}} “.” } {}

Подставляя упрощенные обозначения для ΔxΔx и ΔtΔt, получаем

2,27 v- = x − x0t. v- = x − x0t. размер 12 {{bar {v}} = {{x – x rSub {size 8 {0}}} больше {t}} “.” } {}

Решение для размера xx 12 {x} {} дает

2.28 x = x0 + vt, x = x0 + vt, размер 12 {x = x rSub {size 8 {0}} + {bar {v}} t “” \ (“константа a” \),} {}

где средняя скорость

2.29 v- = v0 + v2 (постоянная a). V- = v0 + v2 (постоянная a).размер 12 {{bar {v}} = {{v rSub {size 8 {0}} + v} больше {2}} “” \ (“constant” a \) “.” } {}

Уравнение v- = v0 + v2v- = v0 + v2 размер 12 {{bar {v}} = {{v rSub {size 8 {0}} + v} over {2}} {} отражает тот факт, что при постоянном ускорении vv size 12 {v} {} – это просто среднее значение начальной и конечной скоростей. Например, если вы постоянно увеличиваете скорость, то есть с постоянным ускорением, с 30 до 60 км / ч, тогда ваша средняя скорость во время этого постоянного увеличения составляет 45 км / ч.Используя уравнение v- = v0 + v2v- = v0 + v2, размер 12 {{bar {v}} = {{v rSub {size 8 {0}} + v} over {2}}} {}, чтобы проверить это, мы видим, что

2.30 v- = v0 + v2 = 30 км / ч + 60 км / ч3 = 45 км / ч, v- = v0 + v2 = 30 км / ч + 60 км / ч3 = 45 км / ч, размер 12 {{bar { v}} = {{v rSub {size 8 {0}} + v} больше {2}} = {{“30 км / ч” + “60 км / ч”} больше {2}} = “45 км / h, “} {}

, что кажется логичным.

Пример 2.8. Расчет смещения: как далеко пробегает бегунок?

Бегун бежит по прямому участку дороги со средней скоростью 4.00 м / с в течение 2,00 мин. Какова его конечная позиция, если исходная позиция равна нулю?

Стратегия

Нарисуйте эскиз.

Окончательная позиция xx размер 12 {x} {} определяется уравнением

2.31 х = х0 + v-t. Х = x0 + v-t. размер 12 {x = x rSub {размер 8 {0}} + {bar {v}} t} {}

Чтобы найти x, x, размер 12 {x} {}, мы определяем значения x0, x0, size 12 {x rSub {size 8 {0}}} {} vv- size 12 {{bar {v}} } {} и tt size 12 {t} {} из постановки задачи и подставьте их в уравнение.

Решение

1. Определите известные. v- = 4,00 м / с, v- = 4,00 м / с, Δt = 2,00 мин, Δt = 2,00 мин, размер 12 {Δt = 2 “.” “00 мин”} {} и x0 = 0 м. X0 = 0 м. size 12 {x rSub {size 8 {0}} = “0 м”} {}

2. Введите известные значения в уравнение.

2.32 x = x0 + v-t = 0 + 4,00 м / с 120 с = 480 м. x = x0 + v-t = 0 + 4,00 м / с 120 с = 480 м. размер 12 {x = x rSub {size 8 {0}} + {bar {v}} t = 0 + left (4 “.” “00 м / с” справа) слева (“120 s” справа) = “480 м “} {}

Обсуждение

Скорость и конечное смещение положительны, что означает, что они направлены в одном направлении.

Уравнение x = x0 + v-tx = x0 + vt size 12 {x = x rSub {size 8 {0}} + {bar {v}} t} {} дает представление о взаимосвязи между смещением, средней скоростью и и время. Это показывает, например, что перемещение является линейной функцией средней скорости. Под линейной функцией мы подразумеваем, что смещение зависит от vv- размера 12 {{bar {v}}} {} , а не от vv- size 12 {{bar {v}}} {} , возведенного в некоторую другую степень. , например v-2.v-2. размер 12 {{bar {v}} rSup {size 8 {2}}} {} На графике линейные функции выглядят как прямые линии с постоянным наклоном.Например, в автомобильной поездке мы проедем вдвое дальше за заданный промежуток времени, если в среднем мы получим 90 км / ч, чем если бы мы получили в среднем 45 км / ч.

Рис. 2.39 Между смещением и средней скоростью существует линейная зависимость. В течение заданного времени t, t, размера 12 {t} {} объект, движущийся в два раза быстрее другого объекта, будет перемещаться вдвое дальше другого объекта.

Решение для окончательной скорости

Мы можем вывести еще одно полезное уравнение, манипулируя определением ускорения.

Подстановка упрощенных обозначений для ΔvΔv и ΔtΔt дает

2,34 a = v − v0t (constanta) .a = v − v0t (constanta). размер 12 {a = {{v – v rSub {size 8 {0}}} больше {t}} “” \ (“constant” a \) “.” } {}

Решение для размера vv 12 {v} {} дает

2.35 v = v0 + at (constanta). V = v0 + at (constanta). размер 12 {v = v rSub {размер 8 {0}} + курсив “at” “” \ (“константа” a \) “.” } {}

Пример 2.9 Расчет конечной скорости: самолет замедляется после приземления

Самолет приземляется с начальной скоростью 70.0 м / с, а затем замедляется со скоростью 1,50 м / с 21,50 м / с2 размер 12 {1 “.” “50 м / с” rSup {размер 8 {2}}} {} на 40,0 с. Какова его конечная скорость?

Стратегия

Нарисуйте эскиз. Мы рисуем вектор ускорения в направлении, противоположном вектору скорости, потому что самолет замедляется.

Решение

1. Определите известные. v0 = 70,0 м / с, v0 = 70,0 м / с, размер 12 {Δv = “70” “.” “0 м / с”} {} a = -1,50 м / с2, a = -1.50 м / с2, размер 12 {a = – 1 “.” “50 м / с” rSup {размер 8 {2}}} {} t = 40.0s.t = 40.0s.

2. Определите неизвестное. В данном случае это конечная скорость vf.vf. размер 12 {v rSub {размер 8 {f}}} {}

3. Определите, какое уравнение использовать. Мы можем рассчитать конечную скорость, используя уравнение v = v0 + at. V = v0 + at. размер 12 {v = v rSub {размер 8 {0}} + курсив “at”} {}

4. Подставьте известные значения и решите.

2.36 v = v0 + at = 70,0 м / с + −1,50 м / с 240,0 с = 10,0 м / с. v = v0 + at = 70,0 м / с + −1.50 м / с 240,0 с = 10,0 м / с. размер 12 {v = v rSub {size 8 {0}} + ital “at” = “70” “.” “0 м / с” + влево (- 1 “.” “50 м / с” rSup {размер 8 {2}} вправо) влево (“40” “.” “0 с” справа) = “10” “. ” “0 м / с”} {}

Обсуждение

Конечная скорость намного меньше начальной скорости, желательно при замедлении, но все же положительная. С помощью реактивных двигателей обратная тяга могла поддерживаться достаточно долго, чтобы остановить самолет и начать движение назад. На это указывает отрицательная конечная скорость, чего здесь нет.

Рис. 2.41 Самолет приземляется с начальной скоростью 70,0 м / с и замедляется до конечной скорости 10,0 м / с, прежде чем направиться к терминалу. Обратите внимание, что ускорение отрицательное, потому что его направление противоположно его скорости, которая положительна.

Помимо полезности при решении задач, уравнение v = v0 + atv = v0 + при размере 12 {v = v rSub {size 8 {0}} + курсив “at”} {} дает нам представление о взаимосвязях между скорость, ускорение и время.Из него видно, например, что

  • конечная скорость зависит от того, насколько велико ускорение и как долго оно длится,
  • , если ускорение равно нулю, то конечная скорость равна начальной скорости (v = v0) (v = v0) размер 12 {\ (v = v rSub {size 8 {0}} \)} {}, как и ожидалось ( т.е. скорость постоянна) и
  • , если aa size 12 {a} {} отрицательно, то конечная скорость меньше начальной.

Все эти наблюдения соответствуют нашей интуиции, и всегда полезно исследовать основные уравнения в свете нашей интуиции и опыта, чтобы убедиться, что они действительно точно описывают природу.

Установление соединений: соединение в реальном мире

Рис. 2.42 Космический шаттл Endeavour стартует из Космического центра Кеннеди в феврале 2010 г. (Фото: Мэтью Симантов, Flickr)

Межконтинентальная баллистическая ракета (МБР) имеет большее среднее ускорение, чем космический шаттл, и достигает большего скорость в первые минуты или две полета – фактическое время горения межконтинентальной баллистической ракеты классифицируется: ракеты с коротким временем горения труднее уничтожить противнику.Но космический шаттл получает большую конечную скорость, так что он может вращаться вокруг Земли, а не сразу возвращаться вниз, как это делает межконтинентальная баллистическая ракета. Космический шаттл делает это за счет более длительного ускорения.

Решение для конечного положения, когда скорость не постоянна (a ≠ 0a ≠ 0)

Мы можем объединить приведенные выше уравнения, чтобы найти третье уравнение, которое позволяет нам вычислить окончательное положение объекта, испытывающего постоянное ускорение. Начнем с

2.37 v = v0 + ат.v = v0 + at. размер 12 {v = v rSub {размер 8 {0}} + курсив “at”} {}

Добавление v0v0 размера 12 {v rSub {size 8 {0}}} {} к каждой стороне этого уравнения и деление на два дает

2.38 v0 + v2 = v0 + 12at. v0 + v2 = v0 + 12at. размер 12 {{{v rSub {size 8 {0}} + v} over {2}} = v rSub {size 8 {0}} + {{1} over {2}} ital “at” “”. } {}

Так как v0 + v2 = v-v0 + v2 = v- size 12 {{{v rSub {size 8 {0}} + v} over {2}} = {bar {v}}} {} для постоянного ускорения, затем

2.39 v- = v0 + 12ат.v- = v0 + 12ат. размер 12 {{bar {v}} = v rSub {size 8 {0}} + {{1} over {2}} ital “at” “.” } {}

Теперь мы подставляем это выражение для vv- size 12 {{bar {v}}} {} в уравнение для смещения, x = x0 + vt, x = x0 + vt, size 12 {x = x rSub {size 8 { 0}} + {bar {v}} t} {}, что дает

2.40 x = x0 + v0t + 12at2 (постоянная a). X = x0 + v0t + 12at2 (постоянная a). size 12 {x = x rSub {size 8 {0}} + v rSub {size 8 {0}} t + {{1} over {2}} ital “at” rSup {size 8 {2}} “” \ ( “константа” а \) “.” } {}

Пример 2.10 Расчет смещения ускоряющегося объекта: Dragsters

Драгстеры

могут развивать среднее ускорение 26,0 м / с и 2,26,0 м / с2. размер 12 {“26” “.” “0 м / с” rSup {size 8 {2}}} {} Предположим, такой драгстер ускоряется из состояния покоя с такой скоростью в течение 5,56 с. Как далеко он пролетит за это время?

Рис. 2.43 Пилот Top Fuel в армии США Тони Sarge Schumacher начинает гонку с контролируемым выгоранием. (Источник: подполковник Уильям Термонд, фото любезно предоставлено U.С. Армия)

Стратегия

Нарисуйте эскиз.

Нас просят найти смещение, которое равно xx, если принять x0x0 size 12 {x rSub {size 8 {0}}} {} равным нулю. Думайте об этом как о стартовой линии гонки. Он может быть где угодно, но мы называем его 0 и измеряем все остальные позиции относительно него. Мы можем использовать уравнение x = x0 + v0t + 12at2x = x0 + v0t + 12at2 size 12 {x = x rSub {size 8 {0}} + v rSub {size 8 {0}} t + {{1} over {2 }} ital “at” rSup {size 8 {2}}} {} , как только мы определим v0, v0, size 12 {v rSub {size 8 {0}}} {} a, a, size 12 {a} { } и размер tt 12 {t} {} из постановки задачи.

Решение

1. Определите известные. Начало из состояния покоя означает, что v0 = 0, v0 = 0, размер 12 {v rSub {size 8 {0}} = 0} {} aa размер 12 {a} {} задается как 26,0 м / с226,0 м / s2 размер 12 {“26” “.” 0` “m / s” rSup {size 8 {2}}} {} и tt size 12 {t} {} дается как 5,56 с.

2. Подставьте известные значения в уравнение, чтобы найти неизвестный размер xx 12 {x} {}.

2.41 x = x0 + v0t + 12at2.x = x0 + v0t + 12at2. size 12 {x = x rSub {size 8 {0}} + v rSub {size 8 {0}} t + {{1} over {2}} ital “at” rSup {size 8 {2}} “.”} {}

Поскольку начальное положение и скорость равны нулю, это упрощается до

2.42 x = 12at2.x = 12at2. размер 12 {x = {{1} больше {2}} ital “at” rSup {size 8 {2}} “.” } {}

Подстановка идентифицированных значений размера aa 12 {a} {} и размера tt 12 {t} {} дает

2,43 x = 1226,0 м / с25,56 с2, x = 1226,0 м / с25,56 с2, размер 12 {x = {{1} больше {2}} влево (“26” “.” “0 м / с” rSup { размер 8 {2}} справа) слева (5 дюймов. “56 с” справа) rSup {размер 8 {2}},} {}

дает

2.44 х = 402 м. Х = 402 м. размер 12 {x = “402 м”} {}

Обсуждение

Если мы переведем 402 м в мили, мы обнаружим, что пройденное расстояние очень близко к четверти мили: стандартное расстояние для дрэг-рейсинга. Так что ответ разумный. Это впечатляющее смещение всего за 5,56 с, но первоклассные драгстеры могут проехать четверть мили даже за меньшее время.

Что еще мы можем узнать, исследуя уравнение x = x0 + v0t + 12at2? X = x0 + v0t + 12at2? размер 12 {x = x rSub {размер 8 {0}} + v rSub {размер 8 {0}} t + {{1} больше {2}} ital “at” rSup {size 8 {2}}} {} Мы видите, что смещение

  • зависит от квадрата прошедшего времени, когда ускорение не равно нулю.В Примере 2.10 драгстер преодолевает только четверть общего расстояния за первую половину прошедшего времени, а
  • , если ускорение равно нулю, то начальная скорость равна средней скорости (v0 = v-v0 = v- size 12 {v rSub {size 8 {0}} = {bar {v}}} {}) и x = x0 + v0t + 12at2x = x0 + v0t + 12at2 size 12 {x = x rSub {size 8 {0}} + v rSub {size 8 {0}} t + {{1} over {2}} ital “at” rSup {size 8 {2}}} {} становится x = x0 + v0t.x = x0 + v0t. размер 12 {x = x rSub {размер 8 {0}} + v rSub {размер 8 {0}} t} {}

Решение окончательной скорости, когда скорость не постоянна (a ≠ 0a ≠ 0)

Четвертое полезное уравнение может быть получено путем другой алгебраической обработки предыдущих уравнений.

Если мы решим v = v0 + atv = v0 + для размера 12 {v = v rSub {size 8 {0}} + ital “at”} {} для t, t, размера 12 {t} {}, мы получим

2,45 t = v − v0a.t = v − v0a. размер 12 {t = {{v – v rSub {размер 8 {0}}} больше {a}} “.” } {}

Подставляя это и v- = v0 + v2v- = v0 + v2, размер 12 {{bar {v}} = {{v rSub {size 8 {0}} + v} over {2}} {} в x = x0 + vt, x = x0 + vt, размер 12 {x = x rSub {size 8 {0}} + {bar {v}} t} {} получаем

2.46 v2 = v02 + 2ax − x0 (константа). V2 = v02 + 2ax − x0 (константа). размер 12 {v rSub {размер 8 {2}} = v rSub {размер 8 {0}} rSup {размер 8 {2}} + 2a слева (x – x rSub {размер 8 {0}} справа) “” \ (“константа” а \) “.”} {}

Пример 2.11 Расчет конечной скорости: Dragsters

Рассчитайте окончательную скорость драгстера в Примере 2.10 без использования информации о времени.

Стратегия

Нарисуйте эскиз.

Уравнение v2 = v02 + 2a (x − x0) v2 = v02 + 2a (x − x0) идеально подходит для этой задачи, поскольку оно связывает скорости, ускорение и смещение и не требует информации о времени.

Решение

1.Определите известные значения. Мы знаем, что v0 = 0, v0 = 0, размер 12 {v rSub {size 8 {0}} = 0} {}, поскольку драгстер запускается из состояния покоя. Затем мы замечаем, что x − x0 = 402 mx − x0 = 402 м размер 12 {x – x rSub {size 8 {0}} = “402 m”} {} (это был ответ в примере 2.10). Наконец, было дано среднее ускорение a = 26,0 м / с2. A = 26,0 м / с2. размер 12 {a = “26” “.” “0 м / с” rSup {size 8 {2}}} {}

2. Подставьте известные значения в уравнение v2 = v02 + 2a (x − x0) v2 = v02 + 2a (x − x0) и решите для vv

2,47 v2 = 0 + 226,0 м / с 2402 м.v2 = 0 + 226,0 м / с 2402 м. размер 12 {v rSup {размер 8 {2}} = 0 + 2 слева (“26” “.” “0 м / с” rSup {размер 8 {2}} справа) слева (“402 м” справа)} { }

Таким образом,

2,48 v2 = 2,09 × 104 м2 / с2. V2 = 2,09 × 104 м2 / с2. размер 12 {v rSup {размер 8 {2}} = 2 “.” “09” умножить на “10” rSup {размер 8 {4}} `m rSup {размер 8 {2}}” / s “rSup {размер 8 {2}}} {}

Чтобы получить v, v, размер 12 { v} {} извлекаем квадратный корень.

2,49 v = 2,09 × 104 м2 / с2 = 145 м / с. V = 2,09 × 104 м2 / с2 = 145 м / с.

Обсуждение

Ответ 145 м / с составляет около 522 км / ч или около 324 миль / ч, но даже эта головокружительная скорость не достигает рекорда для четверти мили.Также обратите внимание, что квадратный корень имеет два значения; мы взяли положительное значение, чтобы указать скорость в том же направлении, что и ускорение.

Исследование уравнения v2 = v02 + 2a (x − x0) v2 = v02 + 2a (x − x0) size 12 {v rSup {size 8 {2}} = v rSub {size 8 {0}} rSup { размер 8 {2}} + 2a \ (x – x rSub {size 8 {0}} \)} {} может дать дальнейшее понимание общих отношений между физическими величинами:

  • Окончательная скорость зависит от того, насколько велико ускорение есть и расстояние, на котором он действует.
  • При фиксированном замедлении автомобиль, который едет вдвое быстрее, не просто останавливается на удвоенном расстоянии – для остановки требуется гораздо больше времени. Вот почему у нас есть зоны с пониженной скоростью возле школ.

Постоянная скорость в сравнении с постоянным ускорением – Когда B догонит A?

Возможно, лучший способ визуализировать это на графике скорость-время. Для этого нам нужно разобрать вопрос.

#color (красный) «Когда я (Стив Дж.) Редактировал этот ответ, чтобы исправить ошибку» #
#color (красный) «Я не изучал и не вносил изменения в график.-2) #
– # цветная (красная) “кошка” # сразу начинает движение, при # t = 0 #. Да, вопрос действительно говорит «точно после», но это значит, что мы не говорим, что кошка ест мышь в момент t = 0, поэтому мы закончили.

Сказав это, мы можем нарисовать график.

Пусть # цвет (синий) T # будет временем, которое # цвет (красный) “кот” # поймал # цвет (зеленый) “мышь” #

Мы можем использовать уравнения движения, чтобы найти уравнения движения как для кошки, так и для мыши.

Оставить комментарий